ca week 7 & 8 - iasbaba

70
IASBABA 60 DAY PLAN 2020 –CURRENT AFFAIRS WEEK 7 AND 8 60 DAYS PROGRAMME-2020 IASBABA

Upload: others

Post on 25-Nov-2021

1 views

Category:

Documents


0 download

TRANSCRIPT

IASBABA60DAYPLAN2020–CURRENTAFFAIRSWEEK7AND8

60DAYSPROGRAMME-2020 IASBABA

IASBABA60DAYPLAN2020–CURRENTAFFAIRSWEEK7AND8

60DAYSPROGRAMME-2020 IASBABA

Q.1)WithreferencetoGlobalConsortiumforGovernanceofDigitalCurrency,considerthefollowingstatements:

1. The consortium will aim for increasing access to the financial system throughinclusive,innovative,andinter-operablepolicysolutions.

2. ItislaunchedattheG20SummitheldatRiyadh.Whichofthestatementsgivenaboveis/arecorrect?

a) 1onlyb) 2onlyc) Both1and2d) Neither1nor2

Q.1)Solution(a)Statement1 Statement2Correct IncorrectTheGlobal Consortium for Digital Currency Governance willaim for increasing access to the financial system throughinclusive, innovative, and inter-operable policy solutions.Initiative aims to bring together leading companies, financialinstitutions, government representatives, technical experts,academics, internationalorganizations,NGOsandmembersoftheForum'scommunitiesonagloballevel.

The World EconomicForum (WEF) hasannounced the firstglobal consortiumfocused on digitalcurrency governanceincludingthestablecoins.

Q.2)Whichof the followingConstitutionalAmendmentsmodifiedArticle312 to includeanAllIndiaJudicialService?

a) 24thConstitution(Amendment)Actb) 38thConstitution(Amendment)Actc) 42ndConstitution(Amendment)Actd) 44thConstitution(Amendment)Act

Q.2)Solution(c)

• After the Swaran Singh Committee’s recommendations in 1976, Article 312 wasmodifiedbythe42ndconstitution (amendment)act in1977toprovide foranAllIndiaJudicialService(AIJS).

IASBABA60DAYPLAN2020–CURRENTAFFAIRSWEEK7AND8

60DAYSPROGRAMME-2020 IASBABA

• AllIndiaJudicialService(AIJS)aimsatcreatingacentralizedcadreofDistrictJudgeswho will be recruited centrally through an all-India examination and allocated toeachStatealongthelinesoftheAllIndiaServices(AIS).

Q.3)Considerthefollowingpairs:River Tributaryof

1. Indravati Mahanadi2. Ghataprabha Krishna3. Kabini Cauvery

Whichofthepairsgivenabovearecorrectlymatched?

a) 1and2onlyb) 3onlyc) 2and3onlyd) 1,2and3

Q.3)Solution(c)

• TheGhataprabhaRiverisatributaryofKrishnaRiverthatflowsinKarnataka.• TheIndravatiRiverisastreamoftheriverGodavari.• The Kabini River is one of themajor tributaries of the river Cauvery in southern

India.Q.4)YueluProclamationaimstoprotectandpromotewhichofthefollowing?

a) Humanrightsb) Linguisticdiversityc) IntellectualPropertyd) PublicHealth

Q.4)Solution(b)

• Yuelu Proclamation is a document on protection and promotion of the world'slinguisticdiversity.

• Yuelu Proclamationwas adopted at the first international conferenceon languageresources protection in Changsha, Central China's Hunan province in 2018byUNESCO.

IASBABA60DAYPLAN2020–CURRENTAFFAIRSWEEK7AND8

60DAYSPROGRAMME-2020 IASBABA

Q.5)India’sfirstSuperFabLabwasinauguratedinwhichofthefollowingcities?

a) Puneb) Hyderabadc) Bengalurud) Kochi

Q.5)Solution(d)

• OnJanuary25,2020,India’sfirstSuperFabLabwas inaugurated inKochi,Kerala.The laboratory is to function in collaboration with Massachusetts Institute ofTechnology(MIT),USA.

• ItistheonlylaboratorythatisbeinginstalledoutsideUS.TheSuperFabLabisbeinginstalledincollaborationwithKSUM(KeralaStartupMission).

Q.6)Whichofthefollowingpairsis/arecorrectlymatched?

1. PatolaSaree–Punjab2. DhanuJatra–Odisha3. LaiHaroba–Tripura

Selectthecorrectstatementsa) 1and2b) 2and3c) 1and3d) Alloftheabove

Q.6)Solution(b)PatolaSaree–GujaratDhanuJatra–OdishaLaiHaroba–TripuraQ.7)VakatakaDynastywasspreadoverwhichofthefollowingstates?

1. AndhraPradesh2. Maharashtra3. MadhyaPradesh4. Gujarat5. Chhattisgarh

Selectthecorrectcode:a) 1,2,3and5

IASBABA60DAYPLAN2020–CURRENTAFFAIRSWEEK7AND8

60DAYSPROGRAMME-2020 IASBABA

b) 2and3c) 2,3and4d) Alloftheabove

Q.7)Solution(d)TheVakatakabranchofthedynastycontinuedtilltheendofthereignofPrithvishenaIIin480 CE. Since no son or daughter of this king is known to have succeeded him, theleadershippassedontoKingHarishenaoftheBashimbranch.BythetimeHarishenadiedin510CE,theVakatakaempirewasatitszenith–coveringAndhra,MaharashtraandmostofMadhya Pradesh. In addition, its influence extended to Konkan, Gujarat, Malwa andChhattisgarh.ThedynastywasevenlargerthanithadbeenunderPravarasenaI.DO READ THIS - https://www.firstpost.com/living/the-vakatakas-a-forgotten-empire-tracing-the-history-of-a-once-powerful-kingdom-that-mysteriously-faded-away-6814501.htmlQ.8)‘ServantsofthePeopleSociety’wasfoundedby

a) LalaLajpatRaib) LokmanyaTilakc) VinobaBhaved) LalaHansraj

Q.8)Solution(a)In 1921, he founded Servants of the People Society, a non-profitwelfare organisation, inLahore,which shifted itsbase toDelhi afterpartition, andhasbranches inmanypartsofIndia.Q.9)‘DasarathiSatakam’waswrittenby

a) KancharlaGopannab) TallapakaAnnamayyac) Tyagarajad) Kshetrayya

Q.9)Solution(a)Dasarathi Satakam is a Telugu Bhakti Satakam, a popular form of Telugu poetry. It waswrittenbyBhaktaRamadasu(KancharlaGopanna)during17thcentury.It consists of 104 poems. Dasarathi means son of Dasaratha, who is the incarnation ofVishnu,Rama.Q.10)Considerthefollowingstatementswithrespectto‘Sennaspectabilis’.

1. IthasbecomeaninvasivealienspeciesofIndia.

IASBABA60DAYPLAN2020–CURRENTAFFAIRSWEEK7AND8

60DAYSPROGRAMME-2020 IASBABA

2. ItisnativetoSouthandCentralAmerica.

Selectthecorrectstatementsa) 1Onlyb) 2Onlyc) Both1and2d) Neither1nor2

Q.10)Solution(c)Senna spectabilis is a plant species of the legume family (Fabaceae) in the subfamilyCaesalpinioideaenativetoSouthandCentralAmerica.IthasbecomeaninvasivespeciesintheNilgiriBiosphereReserve.Invasivespeciesintheregion-eucalyptus,teaplantationsandwattleandnaturalisedalienspecies like Lantana camara, Opuntia stricta, Chromolaena odorata, PartheniumhysterophorusandSennaspectabilisQ.11) With reference to National Strategy for Financial Inclusion (NSFI), consider thefollowingstatements:

1. ThestrategyaimstostrengthentheecosystemforvariousmodesofdigitalfinancialservicesinallTier-IItoTierVIcenters.

2. ItisreleasedbyNITIAayog.3. TomakethePublicCreditRegistryfullyoperationalby2022isoneoftheobjectives

ofthestrategy.Whichofthestatementsgivenaboveis/areINCORRECT?

a) 1and2onlyb) 2onlyc) 2and3onlyd) 1and3only

Q.11)Solution(b)Statement1 Statement2 Statement3Correct Incorrect CorrectThe strategy aims tostrengthen the ecosystemforvariousmodesofdigitalfinancialservicesinallTier-II to Tier VI centers and to

National Strategyfor FinancialInclusion (NSFI) isreleased byReserve Bank of

Someofotherobjectives inNSFI:Everyadult had access to a financial serviceprovider through a mobile device byMarch 2024; Every willing and eligibleadult,whohasbeenenrolledunder the

IASBABA60DAYPLAN2020–CURRENTAFFAIRSWEEK7AND8

60DAYSPROGRAMME-2020 IASBABA

create the necessaryinfrastructure to movetowards a less-cash societybyMarch2022.

India(RBI)fortheperiod2019-2024.

PM JanDhanYojana,beenrolledunderan insurance scheme and a pensionscheme by March 2020; To make thePublic Credit Registry fully operationalbyMarch2022.

Q.12)TheOneTrillionTreesInitiativewaslaunchedrecentlybywhichofthefollowing?

a) FoodandAgricultureOrganization(FAO)b) TheUnitedNationsConventiontoCombatDesertification(UNCCCD)c) TheInternationalUnionforConservationofNature(IUCN)d) WorldEconomicForum(WEF)

Q.12)Solution(d)

• TheOneTrillionTrees Initiative is launchedbyWorldEconomicForum in its2020meetingtogrow,restoreandconserve1trilliontreesacrosstheworldby2030.

• It isalsoaimedatunitinggovernments,NGOs,businessesand individuals inmass-scalenaturerestoration.

• Theglobalinitiativeisaimedatrestoringbiodiversityandhelpfightclimatechange.Q.13) IndiasignedtheMigrationandMobilityPartnershipAgreementwithwhichofthefollowing?

a) Russiab) Francec) Japand) Canada

Q.13)Solution(b)

• MigrationandMobilityPartnershipAgreementbetweenIndiaandFranceaimedatenhancing people-to-people contacts, fostering Mobility of students, academics,researchersandskilledprofessionalsbetweenIndiaandFrance

• Itwillalsohelpinstrengtheningcooperationonissuesrelatedtoirregularmigrationandhumantraffickingbetweenthetwosides.

Q.14)ConsiderthefollowingstatementsaboutTrueNatTest:

IASBABA60DAYPLAN2020–CURRENTAFFAIRSWEEK7AND8

60DAYSPROGRAMME-2020 IASBABA

1. Itisanindigenousmoleculardiagnostictoolfortuberculosisdiagnosis.2. It is a polymerase chain reaction (PCR)-based test which can also detect drug

resistancewiththeuseofchips.Whichofthestatementsgivenaboveis/arecorrect?

a) 1onlyb) 2onlyc) Both1and2d) Neither1nor2

Q.14)Solution(c)Statement1 Statement2Correct CorrectTrueNat Test is anindigenousmolecular diagnostic toolfortuberculosis diagnosis. Itwas developed byGoa-basedMolbioDiagnostics,whichWHOendorsedrecently.

TrueNatisapolymerasechainreaction(PCR)-basedtestthat, by assaying the genes present in the TB bacteria,cannotonlydetectthepresenceofthebacteriabutcanalso detect drug resistance with the use of chips.TrueNat test can easily and cost-effectively be done intheprimaryhealthcaresetup.

Q.15)Considerthefollowingpairs:Buddhistmonasticsites State

1. Moghalmari UttarPradesh2. Bojjannakonda AndhraPradesh3. Lingalametta Karnataka

Whichofthepairsgivenaboveareincorrectlymatched?

a) 1and3onlyb) 1onlyc) 2and3onlyd) 1,2and3

Q.15)Solution(a)

IASBABA60DAYPLAN2020–CURRENTAFFAIRSWEEK7AND8

60DAYSPROGRAMME-2020 IASBABA

• Moghalmari is aBuddhist monasticsite of the early medieval period in WestBengal’sPaschimMedinipurdistrict.

• BojjannakondaandLingalamettaMonastries:ThesearethetwinrockcutBuddhistmonasteriesatSankaramvillageinVisakhapatnam,AndhraPradesh.Theydatebacktothe3rdcenturyBC.

Q.16)Considerthefollowingstatementswithrespectto‘MissionInnovation’.

1. It isaglobal initiativeof24countriesandtheEuropeanUniontoaccelerateglobalcleanenergyinnovation.

2. Itwas launched during the 2015UnitedNations Climate Change Conference (CoP21).

Selectthecorrectstatementsa) 1Onlyb) 2Onlyc) Both1and2d) Neither1nor2

Q.16)Solution(c)Mission Innovation was announced on November 30, 2015 due to pioneering efforts byIndia, France and USA, as world leaders came together in Paris to undertake ambitiousefforts to combat climate changes. Mission Innovation (MI) is a global initiative of 24countries and the European Union to dramatically accelerate global clean energyinnovation.Aspartoftheinitiative,participatingcountrieshavecommittedtodoubletheirgovernments’ cleanenergy researchanddevelopment (R&D) investmentsover fiveyears,whileencouraginggreaterlevelsofprivatesectorinvestmentintransformativecleanenergytechnologies.

IASBABA60DAYPLAN2020–CURRENTAFFAIRSWEEK7AND8

60DAYSPROGRAMME-2020 IASBABA

Q.17)The‘CorruptionPerceptionsIndex’isbroughtoutbythe

a) WorldEconomicForumb) WorldBankc) TransparencyInternational

IASBABA60DAYPLAN2020–CURRENTAFFAIRSWEEK7AND8

60DAYSPROGRAMME-2020 IASBABA

d) InternationalCourtofJustice

Q.17)Solution(c)The Corruption Perceptions Index (CPI) is an index published annually by TransparencyInternational since 1995which ranks countries "by their perceived levels of public sectorcorruption,asdeterminedbyexpertassessmentsandopinionsurveys."Q.18) Which of the following statements is most appropriated with respect to ‘JuiceJacking’.

a) Itisaprocessinwhichtransactionsforvariousformsofcryptocurrencyareverifiedandaddedtotheblockchaindigitalledger.

b) It is a type of cyber-attack involving a charging port that doubles as a dataconnection,typicallyoverUSB.

c) It is the act of pretending to be someone you are not online, in order to luresomeoneyou'venevermetintoarelationship.

d) It is the fraudulent attempt to obtain sensitive information such as usernames,passwordsandcreditcarddetailsbydisguisingoneselfasatrustworthyentityinanelectroniccommunication.

Q.18)Solution(b)Juice jacking is a type of cyber-attack involving a charging port that doubles as a dataconnection, typically over USB. This often involves either installing malware orsurreptitiouslycopyingsensitivedatafromasmartphone,tablet,orothercomputerdevice.Q.19)‘NChandrasekaranCommittee’wasconstitutedtolookinto

a) ArtificialIntelligenceb) 3DPrintingc) BigDatad) AlgorithmTrading

Q.19)Solution(a)NChandrasekaranCommittee.CommitteeconstitutedbyMinistryofDefencetostudyuseandapplicationofartificialintelligence(AI)inmilitary.Q.20)Considerthefollowingstatements

1. Lithium-Sulphur(Li-s)batteriesstoreconsiderablymoreenergythantheLithium-ion(Li-ion)batteries.

2. Li-S batteries are generally considered to be the successors of the Li-ion batteriesbecauseoftheirlowercostofproduction,energyefficiencyandimprovedsafety.

IASBABA60DAYPLAN2020–CURRENTAFFAIRSWEEK7AND8

60DAYSPROGRAMME-2020 IASBABA

Selectthecorrectstatementsa) 1Onlyb) 2Onlyc) Both1and2d) Neither1nor2

Q.20)Solution(c)Theoretically,Lithium-Sulphurbatteriesarecapableofholdinguptofive-timesmoreenergythanLithium-ionones.Li-S batteries are generally considered to be the successors of the Lithium-ion (Li-ion)batteriesbecauseoftheirlowercostofproduction,energyefficiencyandimprovedsafety.Theircostofproductionislowerbecausesulfurisabundantlyavailable.Lithium-ionbatteriesrequiremineralssuchasrareearths,nickelandcobalttoproducetheirpositive electrodes. Supply of these metals is limited, prices are rising, and their miningoftenhasgreatsocialandenvironmentalcosts.Q.21)WhichofthefollowingorganisationreleasedtheWomen,BusinessandTheLawReport2020?

a) OrganisationofEconomicCooperationandDevelopment(OECD)b) UN-DepartmentofEconomicandSocialAffairs(UN-DESA)c) UNWomend) WorldBank

Q.21)Solution(d)

• TheWomen,BusinessandTheLawReport2020isthesixtheditionofthereportbyWorldBanktoanalyzetheimpactoflawsandregulations(legalgenderequality)onwomen’s economic opportunity (entrepreneurship and employment) in 190economies.

• It measures global progress towards gender equality in the law and the legaldifferencesonaccesstoeconomicopportunitiesbetweenmenandwomen.

• Eight indicators are Mobility, Workplace, Pay, Marriage, Parenthood,Entrepreneurship,Assets,andPension.

• Indiawasranked117thandscored74.4outof100.Q.22)Considerthefollowingpairs:

IASBABA60DAYPLAN2020–CURRENTAFFAIRSWEEK7AND8

60DAYSPROGRAMME-2020 IASBABA

CitiesinNews Country1. Al-Asad Iraq2. Sirte Egypt3. Lamu Kenya4. Erbil Syria5. Tripoli Jordan

Whichofthepairsgivenaboveareincorrectlymatched?

a) 2and5onlyb) 1,3and4onlyc) 2,4and5onlyd) 1,2,3and5only

Q.22)Solution(c)

• Al-AsadandErbilmilitarybasesarelocatedinIraq.• LamutownissituatedonthecoastofKenya.• SirteisaCityinLibyalocatedontheMediterraneancoast.• TripoliisinLibya.

Q.23)ConsiderthefollowingstatementsaboutMuggerCrocodile:

1. Itisfoundinfreshwaterhabitatsandalsoincoastalsaltwaterlagoonsandestuaries.2. Itisan‘Endangered’speciesunderIUCNRedList.

Whichofthestatementsgivenaboveis/arecorrect?

a) 1onlyb) 2onlyc) Both1and2d) Neither1nor2

Q.23)Solution(a)Statement1 Statement2Correct IncorrectThe Mugger Crocodile is anegg-layingandhole-nestingspecies. It is mainly restricted to the Indian subcontinent.

IUCN status:Vulnerable. Itis protected under

IASBABA60DAYPLAN2020–CURRENTAFFAIRSWEEK7AND8

60DAYSPROGRAMME-2020 IASBABA

Found in a number offreshwater habitattypes includingrivers, lakes & marshes and also in coastalsaltwaterlagoonsandestuaries.

Schedule Iof theWildlifeProtectionAct,1972.

Q.24)TheZoKutpuiisafestivalofwhichofthefollowingtribes?

a) Mizob) Apatanic) Kukid) Meitei

Q.24)Solution(a)

• ZoKutpui is festivalcelebratedbydifferentMizo tribes isanattempttounifyandstrengthenthebrotherhoodamongvariousMizotribeslivingindifferentpartsoftheworld.

• MizoramgovernmentwillbeorganisingZoKutpui inat least10statesacross IndiaandcountriessuchasUS,MyanmarandBangladesh.

Q.25)With reference to North-East Gas Grid Project, which of these statements is/areNOTcorrect?

1. ThegridconnectsalltheeightstatesinNorth-Easternregion.2. ItisimplementedbyGasAuthorityofIndiaLtd(GAIL).

Selectthecorrectanswerusingthecodegivenbelow:

a) 1onlyb) 2onlyc) Both1and2d) Neither1nor2

Q.25)Solution(b)Statement1 Statement2Correct IncorrectNorth-East Natural Gas Pipeline Grid is the1,656-kmpipelinewhichconnectsGuwahatiinAssam tomajor cities in the region such

The project is critical towards implementingthe government’sHydrocarbon Vision 2030for the North-East. Indradhanush Gas Grid

IASBABA60DAYPLAN2020–CURRENTAFFAIRSWEEK7AND8

60DAYSPROGRAMME-2020 IASBABA

as Itanagar, Dimapur, Kohima, Imphal,AizawlandAgartalacoveringalleightstatesinNorth-Easternregion.

Limited(IGGL)-a jointventurecompanyoffive CPSEs (IOCL, ONGC, GAIL, OIL andNRL)istheimplementationAgency.

Q.26)Considerthefollowingstatements

1. ‘BureauofImmigration(BoI)’isundertheaegisofMinistryofExternalAffairs.2. Thee-FRROschemehasbeenimplementedontheImmigration,VisaandForeigner's

Registration&Tracking(IVFRT)platform.

Selectthecorrectstatementsa) 1Onlyb) 2Onlyc) Both1and2d) Neither1nor2

Q.26)Solution(b)‘BureauofImmigration(BoI)’isundertheaegisofMinistryofHomeAffairs.e-FRROschemeisaimedatbuildingacentralized,transparentonlineplatformfortheforeignerstoavailvisarelatedservicesandtoprovideFaceless,CashlessandPaperlessservicestotheforeignerswithuserfriendlyexperience.Thee-FRROschemehasbeenimplementedontheIVFRTplatformwithnegligibleextraexpenditure,takingacuefromthealreadysuccessfullyrunninge-Visascheme.Q.27)Whichofthefollowingpairsis/arecorrectlymatched?PlacesinNewsCountry

1. Ashdod–Israel2. TaalVolcano–Indonesia3. Hulhumale–SriLanka

Selectthecorrectcodea) 1Onlyb) 1and2c) 2and3d) Alloftheabove

Q.27)Solution(a)Ashdod–IsraelTaalVolcano–PhilippinesHulhumale–MaldivesQ.28)‘'Sahyog-Kaijin'isajointexercisebetweenCoastGuardsofIndiaand

a) Japanb) Chinac) Indonesia

IASBABA60DAYPLAN2020–CURRENTAFFAIRSWEEK7AND8

60DAYSPROGRAMME-2020 IASBABA

d) Thailand

Q.28)Solution(a)Theaimbehind‘Sahyog-Kaijin’istostrengthenthebondbetweenIndiaandJapan.Q.29)Theterm‘ReciprocatingTerritoriesofIndia’wasinnewrecently.Considerthefollowingstatements

1. ThescopeofthetermflowsfromtheCodeofCivilProcedure.2. UnitedArabEmiratesistheonlytobethereciprocatingterritoryofIndia

Selectthecorrectstatementsa) 1Onlyb) 2Onlyc) Both1and2d) Neither1nor2

Q.29)Solution(a)Thedefinition,explanationandscopeofReciprocatingTerritoriesofIndiaflowsfromsection44-AoftheCodeofCivilProcedure,1908(theCode).Abareperusalofsection44-Aof‘thecode’suggeststhatareciprocatingterritorymeansanycountryorterritoryoutsideIndiawhichtheCentralGovernmentmaybynotificationintheofficialgazette,declaretobeareciprocatingterritoryforthepurposeofsection44-A.ApartfromUAE,theothercountriesdeclaredtobe“reciprocatingterritories”are:UnitedKingdom,Singapore,Bangladesh,Malaysia,Trinidad&Tobago,NewZealand,theCookIslands(includingNiue)andtheTrustTerritoriesofWesternSamoa,HongKong,PapuaNewGuinea,Fiji,Aden.Q.30)‘ReskillingRevolution’isaninitiativeby

a) WorldEconomicForumb) WorldBankc) WorldTradeOrganizationd) InternationalMonetaryFund

Q.30)Solution(a)IndiaonWednesdayjoinedasafoundinggovernmentmembertheWorldEconomicForum'sReskillingRevolution,aninitiativetoprovideonebillionpeoplewithbettereducation,skillsandjobsby2030Theschemeaimstofuture-proofworkersfromtechnologicalchangeandhelpeconomiesbyprovidingnewskillsfortheFourthIndustrialRevolution.FoundinggovernmentsincludeBrazil,France,India,Pakistan,theRussianFederation,UAEandtheUS.Q.31)ConsiderthefollowingstatementsaboutBureauofEnergyEfficiency(BEE):

1. ItisastatutorybodyestablishedundertheEnergyConservationAct,2001.

IASBABA60DAYPLAN2020–CURRENTAFFAIRSWEEK7AND8

60DAYSPROGRAMME-2020 IASBABA

2. It developed State Energy Efficiency Index in association with Energy EfficiencyServicesLimited(EESL).

3. ItistheimplementingbodyoftheSmartMeterNationalProgramme.Whichofthestatementsgivenaboveis/arecorrect?

a) 1and2onlyb) 1onlyc) 2and3onlyd) 1,2and3

Q.31)Solution(b)Statement1 Statement2 Statement3Correct Incorrect IncorrectThe Bureau ofEnergy Efficiency(BEE) is a statutorybody under theprovisions of theEnergyConservationAct,2001,undertheMinistryofPower.

The State Energy Efficiency Indextracks the progress ofEnergyEfficiency (EE)initiatives in statesand union territories based on 97significant indicators. The index isdeveloped byBureau of EnergyEfficiency (BEE)in associationwithAlliance for an Energy EfficientEconomy(AEEE).

TheSmart Meter NationalProgrammeis beingimplemented todeploysmartmetersacross the country.The scheme is beingimplementedbyEnergyEfficiency ServicesLimited (EESL), a JV of PSUsunderMinistryofPower.

Q.32)WhichofthefollowingstatementsaboutInternationalMaritimeOrganization(IMO)is/areNOTcorrect?

1. Itistheglobalstandard-settingauthorityforthesafety,securityandenvironmentalperformanceofinternationalshipping.

2. ItistheonlyUNspecialagencytohaveitsheadquartersintheUnitedKingdom.3. IMOmeasurescoverbothaccidentalandoperationaloilpollution.

Selectthecorrectanswerusingthecodegivenbelow:

a) 1onlyb) 2onlyc) 3onlyd) Noneoftheabove

IASBABA60DAYPLAN2020–CURRENTAFFAIRSWEEK7AND8

60DAYSPROGRAMME-2020 IASBABA

Q.32)Solution(d)Statement1 Statement2 Statement3Correct Correct CorrectInternational MaritimeOrganization (IMO) is theglobal standard-settingauthority for the safety,security and environmentalperformance of internationalshipping. Its main role is tocreatearegulatoryframeworkfor the shipping industry thatis fair and effective,universally adopted anduniversallyimplemented.

The objective of IMO isthe improvement ofMaritime safety and theprevention of marinepollution. It is the onlyUN special agency tohave its headquarters inthe United Kingdom,London.Itisthefirsteverinternational organizationdevoted exclusively tomaritimematters.

IMO measures cover allaspects of internationalshippingincludingshipdesign,construction, equipment,manning, operation anddisposal. It covers accidentaland operational oil pollutionalong with different types ofpollutionbychemicals,goodsin packaged form, sewage,garbageandairpollution.

Q.33)Recentlyseeninnews,the‘biorockormineralaccretiontechnology’isrelatedwithwhichofthefollowing?

a) Coralreefrestorationb) Carboncaptureandstoragec) Biominingofmetallicnodulesd) De-contaminatethepollutedwater

Q.33)Solution(a)

• TheZoologicalSurveyof India (ZSI),withhelp fromGujarat’s forestdepartment, isattempting for the first time a process to restore coral reefs using biorock ormineralaccretiontechnologyintheGulfofKachchh.

• Biorock is the name given tothe substance formed by electro accumulation ofmineralsdissolvedinseawateronsteelstructuresthatareloweredontotheseabed

IASBABA60DAYPLAN2020–CURRENTAFFAIRSWEEK7AND8

60DAYSPROGRAMME-2020 IASBABA

and are connected to a power source, in this case solar panels that float on thesurface.

• Fragmentsofbrokencoralsaretiedtothebiorockstructure,wheretheyareabletogrowatleastfourtosixtimesfasterthantheiractualgrowthastheyneednotspendtheirenergyinbuildingtheirowncalciumcarbonateskeletons.

Q.34) The ‘Cyber SafeWomen’initiative launched by the Government of which of thefollowing?

a) UttarPradeshb) Keralac) Maharashtrad) Delhi

Q.34)Solution(c)

• Maharashtra Government launches ‘Cyber Safe Women’initiative. MaharashtraGovernment ‘has launcheda ‘CyberSafeWomen’initiativeunderwhichawarenesscampswillbeheldacrossallthedistrictsofthestateregardingcybersafety.

Q.35)WithreferencetoNilavembukudinee,considerthefollowingstatements:

1. It is an Ayurveda medicine recommended for prevention and management of alltypesofviralinfections/fevers.

2. Itactsasimmunostimulantandimmunomodulator.Whichofthestatementsgivenaboveis/arecorrect?

a) 1onlyb) 2onlyc) Both1and2d) Neither1nor2

Q.35)Solution(b)Statement1 Statement2Incorrect CorrectNilavembukudineer is a Siddha medicinerecommended for prevention andmanagementof

It acts as immunostimulant andimmunomodulator, which boosts

IASBABA60DAYPLAN2020–CURRENTAFFAIRSWEEK7AND8

60DAYSPROGRAMME-2020 IASBABA

all types of viral infections/fevers.It reducessymptomsassociatedwithfever,whichinclude–aheadache, body aches, muscle aches, loss ofenergy,fatigue,weaknessetc.Itisalsoeffectiveforreducing joint pain, joint swelling, muscle pain,headache and rashes associatedwith chikungunyainfection.

immunity and modulates defenceresponse in the body. It containsherbal ingredients, which haveantipyretic, anti-inflammatory,antiviral, and immunomodulatoryactions.

Q.36)Considerthefollowingstatementswithrespectto‘GlobalRefugeeForum’.

1. Thefirst-everGlobalRefugeeForumwasheldinGeneva,Switzerland.2. ItwasorganizedbythegovernmentsofCostaRica,Ethiopia,Germany,andTurkey,

inpartnershipwiththeUNHighCommissionerforRefugees(UNHCR).

Selectthecorrectstatementsa) 1Onlyb) 2Onlyc) Both1and2d) Neither1nor2

Q.36)Solution(c)Thefirst-everGlobalRefugeeForumtookplacefrom17-18December2019inGeneva,Switzerland.TheeventwasorganizedbythegovernmentsofCostaRica,Ethiopia,Germany,PakistanandTurkey,inpartnershipwiththeUNHighCommissionerforRefugees(UNHCR).Itwasco-hostedbythegovernmentofSwitzerland.ThenextGlobalRefugeeForumwilltakeplacein2023,withamid-termreviewmeetingin2021.Q.37)Considerthefollowingstatementswithrespectto‘Carborundum’.

1. Itisasemiconductor2. Itisadesirablemirrormaterialforastronomicaltelescopes.3. Itcanbeusedintheproductionofgraphene.

Selectthecorrectstatementsa) 1and2b) 3Onlyc) 2and3d) 1,2and3

Q.37)Solution(d)Siliconcarbide(SiC),alsoknownascarborundum,isasemiconductorcontainingsiliconandcarbon.Itoccursinnatureastheextremelyraremineralmoissanite.Thelowthermalexpansioncoefficient,highhardness,rigidityandthermalconductivitymakesiliconcarbideadesirablemirrormaterialforastronomicaltelescopes.

IASBABA60DAYPLAN2020–CURRENTAFFAIRSWEEK7AND8

60DAYSPROGRAMME-2020 IASBABA

SiliconcarbidecanbeusedintheproductionofgraphenebecauseofitschemicalpropertiesthatpromotetheepitaxialproductionofgrapheneonthesurfaceofSiCnanostructures.Q.38)‘Yarrabubbacrater’wasinnewsrecently.Whereisitlocated?

a) Australiab) Mexicoc) Mongoliad) Portugal

Q.38)Solution(a)TheYarrabubbacraterisanimpactstructure,theerodedremnantofaformerimpactcrater,situatedinthenorthernYilgarnCratonnearYarrabubbaStationbetweenthetownsofSandstoneandMeekatharra,MidWestWesternAustralia.ItisEarth’soldestknownimpactstructure.Q.39)Whichofthefollowingsectorshasreceivedmaximumallocationunderthe‘NationalInfrastructurePipeline(NIP)’?

a) RenewableEnergyb) UrbanandHousingc) Railwaysd) Roads

Q.39)Solution(d)

Note–Energy(Allincluded)is24%.Q.40)‘NewandEmergingStrategicTechnologies(NEST)’isundertheaegisof

a) MinistryofExternalAffairsb) MinistryofCommerceandIndustry

IASBABA60DAYPLAN2020–CURRENTAFFAIRSWEEK7AND8

60DAYSPROGRAMME-2020 IASBABA

c) MinistryofScienceandTechnologyd) PrimeMinister'sScience,TechnologyandInnovationAdvisoryCouncil

Q.40)Solution(a)TheMinistryofExternalAffairsannouncedthesettingupofNew,EmergingandStrategicTechnologies(NEST)division.Thiswill lookintotheissuespertainingtonewandemergingtechnologies and will help in collaboration with foreign partners in the field of 5G andartificialintelligencewhichareinlinewithIndia’ssecuritygoals.Itsmandateshallinclude,butnotbelimitedto,evolvingIndia’sexternaltechnologypolicyincoordinationwithdomesticstakeholdersandinlinewithIndia’sdevelopmentalprioritiesand national security goals. It will also help assess foreign policy and international legalimplications of new and emerging technologies and technology-based resources, andrecommendappropriateforeignpolicychoice.NEST will negotiate technology governance rules, standards and architecture, suited toIndia’sconditions,inmultilateralandplurilateralframeworks.ItwillalsoundertakecreationofHRcapacitywithintheministryfortechnologydiplomacyworkbyutilisingtheexistingtalent-poolandfacilitatingfunctionalspecialisationofforeignserviceofficersinvarioustechnologydomains.Q.41) Recently RBI has revised the Supervisory Action Framework (SAF) for placingrestrictionsonwhichofthefollowing?

a) ScheduledCommercialBanksb) PaymentBanksc) SmallFinanceBanksd) UrbanCo-operativeBanks

Q.41)Solution(d)

• RBI has revised the Supervisory Action Framework (SAF) for Urban Co-operativeBanks(UCBs).

• SAFissimilartothePromptCorrectiveAction(PCA)frameworkwhichisimposedoncommercialbanks.

• According to revised SAF, UCBs will face restrictions for worsening ofthreeparameters:1. WhennetNon-PerformingAssets(NPAs)exceed6%ofnetadvances,2. When losses are incurred for two consecutive financial years or losses have

accumulatedonthebalancesheets,and3. IfCapitalAdequacyRatio(CAR)fallsbelow9%.

Q.42)Considerthefollowingpairs:

IASBABA60DAYPLAN2020–CURRENTAFFAIRSWEEK7AND8

60DAYSPROGRAMME-2020 IASBABA

Sanctuariesinnews State

1. Dnyanganga UttarPradesh2. Tipeshwar Maharashtra3. Netravali Karnataka

Whichofthepairsgivenabovearecorrectlymatched?

a) 1and2onlyb) 2onlyc) 2and3onlyd) 1,2and3

Q.42)Solution(b)

• WildlifeSanctuariesinnews:§ DnyangangaandTipeshwarWildlifeSanctuary–Maharashtra.§ NetravaliandMhadeiWildlifeSanctuary–Goa.

Q.43)WhichofthefollowingisthehomeportofaircraftcarrierINSVikramaditya?

a) Mumbaib) Karwarc) Cochind) Vishakhapatnam

Q.43)Solution(b)

• INSVikramadityaismodifiedKiev-classaircraftcarrier.Itwasbuiltin1987andhadserved the Soviet navy (named as Baku). It was later renamed Admiral GorshkovundertheRussiannavy.

• The vessel can carry more than 30 long-range multi-role fighters with anti-shipmissiles,air-to-airmissiles,guidedbombsandrockets.

• Itwas retrofittedwith aBarakmissile system (joint developmentwith Israel). It isbasedatitshomeportatKarwarinKarnataka.

Q.44)Considerthefollowingstatements:

1. ZonalCouncilsarethestatutorybodiesestablishedbytheZonalCouncilActof1956.

IASBABA60DAYPLAN2020–CURRENTAFFAIRSWEEK7AND8

60DAYSPROGRAMME-2020 IASBABA

2. Theyareonlydeliberativeandadvisorybodies.3. TheUnionHomeMinisteristhechairmanofallZonalCouncils.

Whichofthestatementsgivenaboveis/arecorrect?

a) 1and3onlyb) 2onlyc) 2and3onlyd) 1,2and3

Q.44)Solution(c)Statement1 Statement2 Statement3Incorrect Correct CorrectZonal Councils are thestatutory (and not theconstitutional) bodiesestablished by StatesReorganization Act of 1956.Theactdividedthecountryintofive zones (Northern, Central,Eastern,WesternandSouthern)andprovidedazonalcouncilforeach zone. ANorth-EasternCouncilwas created by aseparate Act of Parliament,theNorth-Eastern Council Act of1971.

The zonal councils aim atpromoting cooperation andcoordination betweenstates, union territories andtheCentre.Theydiscussandmake recommendationsregarding matters likeeconomic and socialplanning, linguisticminorities, border disputes,inter-state transport, and soon. They are onlydeliberative and advisorybodies.

TheUnionHomeMinisteris the chairman of allZonal Councils. Eachzonal council consists ofthe following members -Union Home Minister;Chief Ministers of all theStates in the zone; twootherministersfromeachstate in the zone andAdministrator of eachunion territory in thezone.

Q.45)ConsiderthefollowingstatementsaboutYellowRust:

1. Itisabacterialdisease.2. Itaffectsthewheatcrop.

Whichofthestatementsgivenaboveis/arecorrect?

a) 1onlyb) 2onlyc) Both1and2d) Neither1nor2

IASBABA60DAYPLAN2020–CURRENTAFFAIRSWEEK7AND8

60DAYSPROGRAMME-2020 IASBABA

Q.45)Solution(b)

• YellowRustisafungaldiseasewhereyellowstripesofpowderordustappearsonleavesandleafsheathsofthewheatcrop.

• Thisoccurswhenthefungalcoloniesintheleavesdrainthecarbohydratesfromtheplant and reduce the green leaf area. The disease affects crop development, andeventuallytheyield(5-30%).

• InIndia,itisamajordiseaseintheNorthernHillZoneandtheNorth-WesternPlainZoneandspreadseasilyduringtheonsetofcoolweatherandwhenwindconditionsarefavorable.Rain,dewandfogfavourthedisease’sdevelopment.

Q.46)Considerthefollowingstatementswithrespectto‘IndianDataRelaySatelliteSystem(IDRSS)’

1. IDRSSwillmaintainacontinuouscommunicationlinkwithIndianremotesensing/earthobservationandothersatellitesinlowearthorbit.

2. IDRSSsatelliteswouldbelaunchedontheGSLVlaunchertogeostationaryorbits.

Selectthecorrectstatementsa) 1Onlyb) 2Onlyc) Both1and2d) Neither1nor2

Q.46)Solution(c)As part of its proposed manned space mission, India will launch the Indian Data RelaySatellite System (IDRSS) to improve data relay and communication links with its remotesensing/earthobservationsatellites.Thetwo-satelliteIDRSSwillmaintainacontinuouscommunicationlinkwithIndia'sremotesensing/earth observation satellites and also with the Geosynchronous Satellite LaunchVehicleMarkIII(GSLVMkIII)thatwouldcarrythreeIndianastronautstospacein2022.Mannedmission apart, IDRSSwillmaintain a continuous communication linkwith Indianremotesensing/earthobservationandothersatellitesinlowearthorbit.The two IDRSS satellites will be placed in geostationary orbit, enabling the satellite tosatellitecommunicationandtransferdata.Q.47)Considerthefollowingstatementswithrespectto‘PyrocumulonimbusCloud’.

1. Itisatypeofcumulonimbuscloudthatformsaboveasourceofheat.2. Thereisnolightningfromsuchclouds.

Selectthecorrectstatementsa) 1Onlyb) 2Onlyc) Both1and2d) Neither1nor2

IASBABA60DAYPLAN2020–CURRENTAFFAIRSWEEK7AND8

60DAYSPROGRAMME-2020 IASBABA

Q.47)Solution(a)Thepyrocumulonimbuscloudsareessentiallyathunderstormthatformsfromthesmokeplumeofafireasintenseheatfromthefirecausesairtoriserapidly,drawingincoolerair.Asthecloudclimbsandthencools inthelowtemperaturesoftheupperatmosphere,thecollisionsoficeparticlesinthehigherpartsofthecloudbuildupanelectricalcharge,whichcanbereleasedaslightning.These can cause dangerous and unpredictable changes in fire behaviour, making themhardertofightaswellascausinglightningstrikesthatcouldignitenewfires.Therisingairalsospursintenseupdraftsthatsuckinsomuchairthatstrongwindsdevelop,causingafiretoburnhotterandspreadfurther.Q.48)Considerthefollowingstatements

1. AccordingtotheConstitution,Parliamentcannotmakelawsonstatesubjects.2. Constitutionobligates theStategovernmenttoensure implementationof the laws

madebytheParliament.3. According to Constitution, Centre is to prevail over the states in the event of any

inconsistencybetweencentralandstatelegislation.

Selectthecorrectstatementsa) 1and2b) 2and3c) 1and3d) Alloftheabove

Q.48)Solution(b)Article 249 empowers Parliament to make laws even on state subjects in the nationalinterest.Under Articles 251 and 254, the Centre is to prevail over the states in the event of anyinconsistencybetweencentralandstatelegislation.Article256oftheConstitutionobligatestheStategovernmenttoensureimplementationofthelawsmadebyParliament.Q.49)Whichofthefollowingisnotdeclareda‘PublicHealthEmergencyofInternationalConcern’accordingtotheWorldHealthOrganisation?

a) 2009H1N1b) 2018NipahVirusc) 2014EbolaVirusd) 1206ZikaVirus

Q.49)Solution(b)APublicHealth Emergencyof International Concern is a formal declarationby theWorldHealthOrganization(WHO)of"anextraordinaryeventwhichisdeterminedtoconstituteapublic health risk to other States through the international spread of disease and topotentially require a coordinated international response", formulated when a situation

IASBABA60DAYPLAN2020–CURRENTAFFAIRSWEEK7AND8

60DAYSPROGRAMME-2020 IASBABA

arises that is "serious, sudden, unusual or unexpected", which "carries implications forpublic health beyond the affected state's national border" and "may require immediateinternationalaction".Since2009therehavebeensixPHEICdeclarations:the2009H1N1(orswineflu)pandemic,the2014poliodeclaration,the2014outbreakofEbolainWesternAfrica,the2015–16Zikavirus epidemic, the ongoing 2018–20 Kivu Ebola epidemic, and the ongoing 2019–20coronaviruspandemic,declaredaPHEICbytheDirector-General.TherecommendationsaretemporaryandrequirereviewseverythreemonthsQ.50)‘SwadeshiSteamNavigationCompany(SSNC)’waslaunchedby

a) V.O.ChidambaramPillaib) S.Ramanathanc) NanaJagannathSunkersettd) MancherjiE.Joshi

Q.50)Solution(a)Read More - https://www.thehindu.com/news/national/tamil-nadu/vo-chidambaram-pillai-s-entire-writings-to-be-published-in-two-volumes/article30552885.eceQ.51)Recently10wetlandsfromIndiawereaddedtothelistof‘WetlandsofInternationalImportance’.WithreferencetoRamsarsitesinIndia,considerthefollowingstatements:

1. Withnewadditions,thetotalRamsarsitesinIndiaincreasedto36.2. UttarPradeshhasthehighestnumberofRamsarsitesthanotherIndianStates.3. AllSouthernStatesexceptGoaandTelanganahasatleastoneRamsarsiteinit.

Whichoftheabovestatementis/areincorrect?

a) 1and2onlyb) 2and3onlyc) 1and3onlyd) 1,2and3

Q.51)Solution(c)

• RamsarConvention is formallyknownasConventiononWetlandsof InternationalImportance,especiallyasWaterfowlHabitat.

• Ramsar Convention has two fold objectives viz. Conservation and sustainableutilizationofwetlands;andstoptheencroachmentandlossofwetlands.

• The10newRamsarsitesare:§ NandurMadhameshwar,afirstforMaharashtra.

IASBABA60DAYPLAN2020–CURRENTAFFAIRSWEEK7AND8

60DAYSPROGRAMME-2020 IASBABA

§ Keshopur-Miani,BeasConservationReserveandNangalinPunjab.§ Nawabganj,ParvatiAgra,Saman,Samaspur,SandiandSarsaiNawarinUttar

Pradesh.Statement1 Statement2 Statement3Incorrect Correct IncorrectIndia has added10morewetlandsto thelistof27RamsarsitesinIndia. With this,a totalof 37 sites in the Indiahave been recognisedunder the internationaltreaty.

Uttar Pradeshwith 7 sites is theState has thehighestnumberofRamsar sites,followedbyPunjabwith6sites.

In southern State, there is noRamsar sitein Karnataka, Goa and Telangana. Keralahas 3 (Ashtamudi Wetland, SasthamkottaLake Vembanad-Kol Wetland), Tamil Nadu(Point Calimere), Maharashtra (NandurMadhameshwar) and Andhra Pradesh(Kolleru)hasoneeach.

Q.52)TheWorldEconomicSituationandProspects(WESP)Reportisaannualpublicationof

a) WorldEconomicForum(WEF)b) OrganisationofEconomicCooperationandDevelopment(OECD)c) UnitedNations(UN)d) WorldBank

Q.52)Solution(c)

• WorldEconomicSituationandProspects (WESP)Report isanannualUN flagshippublicationonthestateoftheworldeconomy,viewedthroughthelensofthe2030AgendaforSustainableDevelopment.

• It is released jointly byUnitedNationsDepartment of Economic and Social Affairs(UN/DESA), UnitedNations Conference on Trade andDevelopment (UNCTAD) andthefiveUnitedNationsregionalcommissions

Q.53)Theterm‘Drosophila’seeninnewsisa

a) DigitalCurrencyb) ModelOrganisminresearchc) InvasiveAlienSpeciesd) ComputerMalware

IASBABA60DAYPLAN2020–CURRENTAFFAIRSWEEK7AND8

60DAYSPROGRAMME-2020 IASBABA

Q.53)Solution(b)

• 5th Edition of Asia Pacific Drosophila Conference was held at Pune. It is beingorganised in the country for the first time by the Indian Institute of ScienceEducationandResearch,Pune.

• It is abiennial conferenceand it aims topromote the interaction of DrosophilaresearchersintheAsia-Pacificregionwiththeirpeersintherestoftheworld.

• Drosophilaisagenusoftwo-wingedfliescommonlyknownasfruitflies.Drosophilais one of the most widely-used and preferred model organisms inbiologicalresearchacrosstheworldforthelast100years.

• Itsgenomeisentirelysequencedandthereisenormousinformationavailableaboutitsbiochemistry,physiologyandbehaviour,

• Approximately 60% of a group of readily identified genes that are mutated,amplified, or deleted in a diverse set of human diseases have a counterpartinDrosophila.

Q.54)ConsiderthefollowingstatementaboutBlueDotNetwork:

1. It isamulti-stakeholder initiativetobringtogethergovernments,theprivatesectorandcivilsociety.

2. It is expected to serveasaglobally recognisedevaluationandcertification systemformarineecosystemrestorationtechniques.

Whichofthestatementsgivenaboveis/arecorrect?

a) 1onlyb) 2onlyc) Both1and2d) Neither1nor2

Q.54)Solution(a)Statement1 Statement2Correct IncorrectBlue Dot Network is amulti-stakeholderinitiativeto bring together governments, theprivate sector and civil society to promote high-quality, trusted standards for global infrastructuredevelopment.TheBDNwasformallyannouncedon4thNovember, 2019 at the Indo-Pacific Business

It is expected to serve as a globallyrecognised evaluation andcertification system for roads, portsandbridgeswithafocusontheIndo-Pacific region. Infrastructureprojectswould be graded on debt,

IASBABA60DAYPLAN2020–CURRENTAFFAIRSWEEK7AND8

60DAYSPROGRAMME-2020 IASBABA

ForuminBangkok,Thailand.ItwillbeledbytheUSalongwithJapanandAustralia.

environmental standards, labourstandardsetc.

Q.55)Considerthefollowingpairs:MultilateralConference Hostcountry

1. Shangri-LaDialogue China2. RaisinaDialogue India3. SagarmathaSambaad Bangladesh

Whichofthepairsgivenaboveareincorrectlymatched?

a) 1onlyb) 1and2onlyc) 3onlyd) 1and3only

Q.55)Solution(d)Pair1 Pair2 Pair3Incorrect Correct IncorrectRaisina Dialogue ofIndia isdesignedonthe lines ofSingapore'sShangri-LaDialogue.

RaisinaDialogue2020 isamultilateralconferencecommitted toaddressingthe most challengingissues facing the globalcommunity, heldannuallyinNewDelhi.

Sagarmatha Sambaad is an initiative oftheGovernment of Nepalto hostapermanent biennial global dialoguestarting from 2020. It is amulti-stakeholderdialogueforumcommittedtodeliberate on themost prominent issuesofglobal, regional and nationalsignificance.

Q.56)‘LaspeyresIndex’isassociatedwith

a) Volatilityinastockmarketb) ConsumerPriceIndexc) UnemploymentduetoRecessiond) Non-PerformingAssets

Q.56)Solution(b)Laspeyres Index isamethodologytocalculatetheconsumerprice indexbymeasuringthechangeinthepriceofthebasketofgoodstothebaseyear.

IASBABA60DAYPLAN2020–CURRENTAFFAIRSWEEK7AND8

60DAYSPROGRAMME-2020 IASBABA

THINK!• Paasche’sIndex

Q.57)The‘ShatrughanChauhanSupremeCourtJudgementisassociatedwith

a) MercyPetitionsb) RighttoMarryc) RighttoPropertyd) RighttoPrivacy

Q.57)Solution(a)In the Shatrughan Chauhan case, the rejection of mercy petitions by the President waschallenged on the grounds of undue delay in disposal of their mercy petitions, mentalillness,andsolitaryconfinementassuperveninggrounds.UnduedelaybythePresidentinrejectingmercytoadeathrowconvictamountstotorture.Such inordinateandunexplaineddelaybythePresident issufficient in itself toentitle theconvicttoacommutation.Q.58)Considerthefollowingstatementswithrespectto‘OrganisationfortheProhibitionofChemicalWeapons(OPCW)’

1. TheOPCWhasthepowertosaywhetherchemicalweaponswereusedinanattackithasinvestigated.

2. ItisanagencyoftheUnitedNationsandhasaseatintheHague,Netherlands.

Selectthecorrectstatementsa) 1Onlyb) 2Onlyc) Both1and2d) Neither1nor2

Q.58)Solution(a)The OPCW, with its 193 member states, has its seat in The Hague, Netherlands, andoversees the global endeavour for the permanent and verifiable elimination of chemicalweapons.TheorganisationisnotanagencyoftheUnitedNations,butcooperatesbothonpolicyandpracticalissues.TheOPCWhasthepowertosaywhetherchemicalweaponswereusedinanattack ithasinvestigated.Q.59)Considerthefollowingstatementswithrespectto‘BiologicalWeaponsConvention’.

1. It prohibits use but not possession or development of chemical and biologicalweapons.

2. ItwasregisteredinLeagueofNationsTreatySeriesin1929.

Selectthecorrectstatementsa) 1Only

IASBABA60DAYPLAN2020–CURRENTAFFAIRSWEEK7AND8

60DAYSPROGRAMME-2020 IASBABA

b) 2Onlyc) Both1and2d) Neither1nor2

Q.59)Solution(d)The Convention on the Prohibition of the Development, Production and Stockpiling ofBacteriological (Biological) and Toxin Weapons and on their Destruction was the firstmultilateraldisarmamenttreatybanningtheproductionofanentirecategoryofweapons.Itwasopenedforsignatureon10April1972andenteredintoforce26March1975Q.60)‘TalChhaparSanctuary’islocatedin

a) Gujaratb) MadhyaPradeshc) Rajasthand) Maharashtra

Q.60)Solution(c)Tal Chhapar Sanctuary is a sanctuary located in the Churu district of NorthwesternRajasthan.Q.61) With reference to National e-Vidhan Application (NeVA) Project, Consider thefollowingstatements:

1. NeVA is mission mode project to digitize the functioning of State Legislatures inIndia.

2. TheMinistryofParliamentaryAffairsisthenodalministryforitsimplementation.

3. AndhraPradeshisthefirstDigitalLegislatureofthecountry.

Whichofthestatementsgivenaboveis/arecorrect?

a) 1and2only

b) 2and3only

c) 1only

d) 1,2and3

Q.61)Solution(a)

• NeVA aims to make all the Legislatures of the country paperless bymaking theproceedingsoftheHousesdigital.

• NeVA is aMissionMode Project (MMP) included in Digital India Programme andMinistry of Parliamentary Affairs (MoPA) is the ‘Nodal Ministry’ for itsimplementationinallthe31States/UTswithLegislatures.

IASBABA60DAYPLAN2020–CURRENTAFFAIRSWEEK7AND8

60DAYSPROGRAMME-2020 IASBABA

• HimachalPradeshisthefirstDigitalLegislatureofthecountry.• Onceimplemented,everyinformationofthelegislativeassemblywouldbeavailable

ine-devicesforthemembersoftheassembly.

• InordertoprovideassistancetotheMembers,aNeVAKendra(e-FacilitationCenter)willbesetupateachlocationunderaNodalOfficer

• FundingofNeVAisonthepatternofCentralSponsoredSchemei.e.60:40;and90:10forNorthEast&hillyStatesand100%forUTs.

• The Secretary (State Legislature) will be the Executing Authority for NeVA inrespectiveStateLegislature.

Q.62)ConsiderthefollowingstatementsaboutMarineOtter

1. Itiscategorisedas‘endangered’bytheIUCN.

2. ItisendemictoSouthAfrica

Whichofthestatementsgivenaboveis/areincorrect?

a) 1only

b) 2only

c) 1and2

d) Neither1nor2

Q.62)Solution(b)

MarineOtteriscategorisedas‘endangered’bytheIUCN.TheMarineOtterhasarestricteddistributionalongthePacificcoastfromnorthernPerualongtheChileancoasttoCapeHornandIsladeLosEstadosinArgentina.

MarineOtterisendemictoSouthAmerica.ItisgenerallyfoundintheregionsofArgentina,ChileandPeru.

• Speciesofmammalsassociatedwithfreshwaterhabitat.

• Huntedfortheirbrush-like,water-resistantfur.

• Unliketheseaotter,whichspendsitswholelifeintheocean,themarineotterlivesonland,feedsalongrockyshores,andmakesonlyshorttripsintothewatertohunt.

Q.63)Whichofthefollowingstatementsis/arecorrectregardingHojagiri?

1. Hojagiriisafolkdance,performedinthestateofNagaland.

2. Thedanceisperformedonlybymen.

Selectthecorrectanswerusingthecodegivenbelow:

IASBABA60DAYPLAN2020–CURRENTAFFAIRSWEEK7AND8

60DAYSPROGRAMME-2020 IASBABA

a) 1only

b) 2only

c) Both1and2

d) Neither1nor2

Q.63)Solution(d)

• Hojagiriisafolkdance,performedinthestateofTripurabytheReang(Bru)people.• Thedanceisperformedbywomenandyounggirls,about4to6membersinateam,

singing,balancingonanearthenpitcherandmanagingotherpropssuchasabottleontheheadandearthenlamponthehandwhileonlythelowerhalfofthebodyismoved.

• ThedanceisperformedontheoccasionoftheHojagirifestivalsandisaccompaniedbybamboo-madeflute,cymbalandkhamb.

Q.64)WithreferencetoRanikiVav,Considerthefollowingstatements:

1. ItislocatedinRajasthan.

2. ItistheonlystepwelltobedeclaredaUNESCOWorldHeritagesite.

3. ItisLocatedonthebanksofSaraswatiriver

Whichofthestatementsgivenaboveis/arecorrect?

a) 1and2only

b) 1and3only

c) 2and3only

d) 1,2and3

Q.64)Solution(c)

• Rani-ki-Vav is an exceptional example of a distinctive form of subterraneanwaterarchitectureoftheIndiansubcontinent,thestepwell,whichislocatedonthebanksoftheSaraswatiRiverinPatan(Gujarat).

• StepwellsinPatan,Gujarat,istheonlystepwelltaggedasaUNESCOWorldHeritagesite.

• Rani(Queen)Udayamaticommissionedthisvavorstepwell,in1063inthememoryofherhusbandKingBhimdevIoftheSolankidynasty.Thevavwaslaterfloodedbythe nearby Saraswati river and silted over until the late 1980s, when it was

IASBABA60DAYPLAN2020–CURRENTAFFAIRSWEEK7AND8

60DAYSPROGRAMME-2020 IASBABA

excavatedby theArcheological Surveyof India,with thecarvings found inpristinecondition.

Q.65)ConsiderthefollowingstatementsaboutGharials

1. Itiscategorisedas‘criticallyendangered’bytheIUCN.

2. Gharialsliveinclearfreshwaterriversystems.

3. GharialsarenowfoundonlyinIndiaandNepal.

Whichofthestatementsgivenaboveis/arecorrect?

a) 1and2only

b) 2and3only

c) 2only

d) 1,2and3

Q.65)Solution(d)

• Itiscategorisedas‘criticallyendangered’bytheIUCN.• Gharialsliveinclearfreshwaterriversystems.• The species is now limited to only 14 widely spaced, restricted localities in north

IndiaandlowlandNepal.• MadhyaPradeshtopingharialcount.

• Reason for increase innumberofgharials in theState is the creationofDevri EcoCentrewherecrocodileeggsarereared.

• India’s first gharial breeding center was built inNandankanan Zoological Park,Bhubaneswar,Odisha.

• NationalChambalSanctuaryistheonlyplaceinIndiawherealargewildpopulationofIndia’sspecialcrocodilianGharialisfound.

Q.66)Considerthefollowingstatements

1. IndianPangolinisfoundinIndiaonly2. ChinesePangolinisfoundinChinaonly

Selectthecorrectstatementsa) 1Onlyb) 2Onlyc) Both1and2d) Neither1nor2

IASBABA60DAYPLAN2020–CURRENTAFFAIRSWEEK7AND8

60DAYSPROGRAMME-2020 IASBABA

Q.66)Solution(d)IndianPangolin(EN)-India;Nepal;Pakistan;SriLankaChinese Pangolin (CR) - Bangladesh; Bhutan; China; Hong Kong; India; Lao People'sDemocraticRepublic;Myanmar;Nepal;Taiwan,ProvinceofChina;Thailand;VietNamQ.67)‘UNHumanRightsCouncilResolution30/1anditstwosuccessorresolutions,34/1and40/1’wereinnewsrecently.Itisassociatedwithwhichofthefollowingcountries?

a) Myanmarb) Chinac) SriLankad) Rwanda

Q.67)Solution(c)Sri Lanka made the ignominious announcement that it does not feel bound by thecommitments made in 2015's UN Human Rights Council Resolution 30/1 and its twosuccessorresolutions,34/1and40/1,intendedtoencouragereformandtransitionaljustice.Q.68)Whichofthefollowingstatementsis/arecorrect?

1. TheessenceofArticle30(1)istoensureequaltreatmentbetweenthemajorityandtheminorityinstitutions.

2. Right of minorities to establish and administer educational institutions of theirchoiceisabsolute.

Selectthecorrectstatementsa) 1Onlyb) 2Onlyc) Both1and2d) Neither1nor2

Q.68)Solution(a)Whenitcomestotherighttoappointteachers, intermsof lawlaiddownintheTMAPaiFoundationcase,aregulationframed inthenational interestmustnecessarilyapplytoallinstitutionsregardlesswhethertheyarerunbymajorityorminorityastheessenceofArticle30(1)istoensureequaltreatmentbetweenthemajorityandminorityinstitutions.Article30(1)(rightofminoritiestoestablishandadministereducationalinstitutionsoftheirchoice)wasneitherabsolutenorabovethelaw.Read More -https://www.newindianexpress.com/magazine/voices/2020/jan/12/supreme-court-delivers-judgment-40-2088167.htmlQ.69) Consider the following statements with respect to ‘International Crops ResearchInstitutefortheSemi-AridTropics(ICRISAT)’

1. ItisheadquarteredinHyderabad.2. Itwasfoundedin1972byaconsortiumoforganisationsconvenedbytheFordand

theRockefellerfoundations.

IASBABA60DAYPLAN2020–CURRENTAFFAIRSWEEK7AND8

60DAYSPROGRAMME-2020 IASBABA

Selectthecorrectstatementsa) 1Onlyb) 2Onlyc) Both1and2d) Neither1nor2

Q.69)Solution(c)The International Crops Research Institute for the Semi-Arid Tropics (ICRISAT) is aninternational organization which conducts agricultural research for rural development,headquartered in Patancheru (Hyderabad, Telangana, India)with several regional centers(Bamako (Mali), Nairobi (Kenya)) and research stations (Niamey (Niger), Kano (Nigeria),Lilongwe(Malawi),AddisAbaba(Ethiopia),Bulawayo(Zimbabwe)).It was founded in 1972 by a consortium of organisations convened by the Ford and theRockefellerfoundations.ItscharterwassignedbytheFAOandtheUNDP.Since its inception, host country India has granted a special status to ICRISAT as a UNOrganizationoperating in the Indianterritorymaking iteligible forspecial immunitiesandtaxprivileges.Q.70)‘TIGR2ESS’isconcernedwith

a) TraffickingofWildAnimalsb) LandDegradationNeutralityc) FoodSecurityd) WetlandsConservation

Q.70)Solution(c)ThefirstGeneralAssemblyofTIGR2ESS(TransformingIndia’sGreenRevolutionbyResearchandEmpowermentforSustainableFoodSupplies),anUK-Indiaresearchprogramme,whichbeganin2018andbroughtover20researchinstitutionstogether.Theprogrammeseeks toaddress four key researchquestions:What shouldanEvergreenRevolution deliver? Can crop productivity increase,whilstmaintaining yield stability? Canwater supplies be shared to match community demand? How can we best engage andeducateforlocalcommunitywell-being?To further strengthen collaboration, specifically in agriculture research, he announced afellowshipprogrammethatwillsoonberolledoutwithIndia’sDepartmentofBiotechnology(DBT) and the National Institute of Plant Genome Research (NIPGR). The fellowship willfacilitate30 Indian researchers toundertake researchworkat crop scienceuniversities intheUKfortwoyears.

IASBABA60DAYPLAN2020–CURRENTAFFAIRSWEEK7AND8

60DAYSPROGRAMME-2020 IASBABA

Q.71) With reference to Soil Health Card (SHC) Scheme, Consider the followingstatements:

1. SHCisaprintedreportthatafarmerwillbehandedoverforeachofhisholdings.

2. Thecostofsampling,testingandreportingisbornebyStateGovernment.

Whichofthestatementsgivenaboveis/arecorrect?

a) 1only

b) 2only

c) Both1and2

d) Neither1nor2

Q.71)Solution(b)

• SHCisaprintedreportthatafarmerwillbehandedoverforeachofhisholdings.Itwillcontainthestatusofhissoilwithrespectto12parameters.

• The cost of sampling, testing and reporting is borne by Central Government. Itprovidesthesumtothestategovernments.

• SHCisaprintedreportthatafarmerwillbehandedoverforeachofhisholdings.Itis provided to all farmers in the country at an interval of 2 years to enable thefarmerstoapplyrecommendeddosesofnutrientsbasedonsoiltestvalues.

• In India, the current consumption of Nitrogen, Phosphorus, and Potassium (NPK)ratio is 6.7:2.4:1, which is highly skewed towards nitrogen (urea) as against idealratioof4:2:1.

• The scheme is being promoted by Department of Agriculture, Cooperation andFarmers welfare under Ministry of Agriculture and Farmers Welfare andimplementedbyrespectiveDepartmentofAgricultureinStates&UTs.

• FarmerscanalsotracksoilsamplesontheSoilHealthCardPortal.

• Soilsamplesaretestedwithrespectto12parameters:

IASBABA60DAYPLAN2020–CURRENTAFFAIRSWEEK7AND8

60DAYSPROGRAMME-2020 IASBABA

• Macronutrients:Nitrogen(N),Phosphorus(P),Potassium(K);

• Secondarynutrient:Sulphur(S);

• Micronutrients:Zinc(Zn),Iron(Fe),Copper(Cu),Manganese(Mn),Boron(B);

• Physicalparameters:pH,EC(electricalconductivity),OC(organiccarbon).

• During 2019-20, a pilot project ‘Development of Model Villages’ has beenundertaken up where soil samples collection has been done at individual farmholdingwithfarmer’sparticipationinsteadofsamplecollectionatgrids.

Q.72)ThetermSUTRAPICissometimesseeninthenewswithreferenceto

a) Enhancing activity of antibiotics and anticancer drugs, and other medicinalproperties.

b) SystematicscientificinvestigationofuniquenessofpureIndigenousIndiancows.

c) Standardizeeffective,economicandeco-friendlypreparationsofUtilityproducts.

d) Performscientificinvestigationsonplantgrowth,soilhealthandprovidingimmunityinplantsystem.

Q.72)Solution(b)

• SUTRA PIC: Scientific Utilisation Through Research Augmentation-Prime ProductsFrom Indigenous Cows. One of the major aims is the systematic scientificinvestigationofuniquenessofpureIndigenousIndiancows.

• Tobefundedbymultiplescientificministries,theinitiativeisledbytheDepartmentofScienceandTechnology(DST).

• CowpathyisatreatmentbasedonproductsobtainedfromIndigenouscowasusedinAyurveda.

SUTRAPICCoversfollowingfivethematicareas:

• UniquenessofIndigenousCows

• Prime-productsfromIndigenousCowsforMedicineandHealth

• Prime-productsfromIndigenousCowsforAgriculturalApplications

• Prime-productsfromIndigenousCowsforFoodandNutrition

• Prime-productsfromindigenouscows-basedutilityitems

Q.73) TheMinistry of New and Renewable Energy (MNRE) aims to set up Ultra MegaRenewableEnergy(RE)Parksat

IASBABA60DAYPLAN2020–CURRENTAFFAIRSWEEK7AND8

60DAYSPROGRAMME-2020 IASBABA

a) MaharashtraandKarnataka

b) GujaratandRajasthan

c) DelhiandHaryana

d) TamilNaduandTelangana

Q.73)Solution(b)

• The Ministry of New and Renewable Energy (MNRE) aims to set up Ultra MegaRenewable Energy (RE) Parks with a capacity of a total of 50 GW in Gujarat andRajasthan.

• The initiative could be the one of the largest renewable energy investmentprogrammesintheworld.

• Khavada inGujaratandJaisalmer inRajasthanhavebeen identifiedforREparksof25,000megawatt(25GW)each.

• TheMNREhasalsorequestedtheMinistryofPowertostrengthenthetransmissioninfrastructuretotheselocationswithintwoyearsfortheevacuationofpowerfromtheseparks.

Q.74) National e-Governance Services Delivery Assessment (NeSDA), 2019 frameworkcoverswhichofthefollowingsixsectors?

1. Labour&Employment

2. Education

3. Transport

4. LocalGovernment&Utilities

5. Insurance

6. Environment

Selectthecorrectanswerusingthecodegivenbelow:

a) 2,3and4only

b) 1,2and4only

c) 1,2,4and6only

d) 1,3,4and5only

Q.74)Solution(c)

IASBABA60DAYPLAN2020–CURRENTAFFAIRSWEEK7AND8

60DAYSPROGRAMME-2020 IASBABA

• National e-Governance Service Delivery Assessment (NeSDA) 2019 rankings werereleasedbyDepartmentofAdministrativeReformsandPublicGrievances.

• Itisdonefor4categories:Unionterritories(7),North-EasternStatesandHillstates(11),Remainingstates(18),Centralgovernmentministrieswebsites.

• It primarily assesses all State/UT and Central Ministry service portals on 7 keyparameters:Accessibility, ContentAvailability, EaseofUse, Information Security&Privacy, End service Delivery, Integrated Service Delivery and Status & RequestTracking.

• Framework covers six sectors - Finance, Labour & Employment, Education, LocalGovernment & Utilities, Social Welfare (including Agriculture & Health) andEnvironment(includingFire).

Ranking

• Delhi, Chandigarh, Daman and Diu administration are the winners among Unionterritoriesacrossallparameters.

• Haryana and Rajasthan are the leading states in the assessment under the"remainingstates"categorycomprising18states.

• ThewebsiteoftheCentralBoardofDirectTaxes(CBDT)undertheFinanceMinistryisthewinnerundertheassessmentofcentralministryserviceportalscategory.

• TheministryportalsofHealth&FamilyWelfare,andHumanResourcehaveemergedleadersacross"allparameters".

• IntermsofportalsKeralascoredthehighestinstatesandUTscategory.

Q.75)Thesignatoryof“AgreementforBringingPeacetoAfghanistan”are

a) RussiaandUSA

b) USAandTaliban

c) PakistanandTaliban

d) IndiaandAfghanistan

Q.75)Solution(b)

• The United States of America signed the “Agreement for Bringing Peace toAfghanistan”withtheTalibaninDoha,Qatar.

• TheUSwill pull its troops out ofAfghanistan after nearly 19 years in return for asecurityguaranteefromtheTalibangroup.

IASBABA60DAYPLAN2020–CURRENTAFFAIRSWEEK7AND8

60DAYSPROGRAMME-2020 IASBABA

• Talibanwilltakesteps“topreventanygrouporindividual,includingal-Qa’ida,fromusing the soil of Afghanistan to threaten the security of theUnited States and itsallies.”

Q.76)Considerthefollowingstatementswithrespectto‘KalinjeEcotourism’

1. It isapartofacommunity-basedconservation initiative for theupliftmentof localcommunitiesandpromotionofmangroveandmarinebiodiversityconservation.

2. ItisaninitiativebytheMaharashtraForestDepartment.

Selectthecorrectstatementsa) 1Onlyb) 2Onlyc) Both1and2d) Neither1nor2

Q.76)Solution(c)KalinjeEcotourism isapartofacommunity-basedconservation initiative takenupby theMangrove and Marine Biodiversity Conservation Foundation of Maharashtra ForestDepartment for the upliftment of local communities and promotion of mangrove andmarinebiodiversityconservation.Q.77) Which of the following species is the keystone species of Nauradehi WildlifeSanctuary?

a) IndianWolfb) Jerdon’sCourserc) BatagurTurtled) AsiaticLion

Q.77)Solution(a)IndianwolfisthekeystonespeciesofNauradehiWildlifeSanctuary.Q.78)Considerthefollowingstatements

1. The Wetland (Conservation and Management) Rules, 2017 removed ‘backwaters,lagoon,creeks,andestuaries’fromthedefinitionof‘wetlands’.

2. Atotalof37sitesinIndiahavebeenrecognisedundertheRamsarConvention.

Selectthecorrectstatementsa) 1Onlyb) 2Onlyc) Both1and2d) Neither1nor2

Q.78)Solution(c)

IASBABA60DAYPLAN2020–CURRENTAFFAIRSWEEK7AND8

60DAYSPROGRAMME-2020 IASBABA

Indiahasadded10morewetlandstositesprotectedbytheRamsarConvention.Withthis,atotalof37sitesinthecountryhavebeenrecognisedundertheinternationaltreaty.NandurMadhameshwar inMaharashtra; Keshopur-Miani, Beas ConservationReserve andNangalinPunjab;andNawabganj,ParvatiAgra,Saman,Samaspur,SandiandSarsaiNawarinUP.The 2010 version of the Rules provided for a CentralWetland Regulatory Authority; the2017Rules replace itwith state-level bodies and createdaNationalWetlandCommittee,which functions inanadvisoryrole.Thenewerregulationsremovedsome items fromthedefinitionof“wetlands”includingbackwaters,lagoon,creeks,andestuaries.Q.79)‘Muktoshri—alsocalledIET21845’isa

a) PortableReverseOsmosiswaterpurifierdevelopedbyDRDO.b) Arsenic-resistantricecultivatedinWestBengal.c) MedicinedevelopedbyMinistryofAyushtotreatDiabetes.d) Noneoftheabove

Q.79)Solution(b)Itisaricevarietythatisresistanttoarsenic.Severalstudieshaveshownthatarsenicfromgroundwaterandthesoilcanenterthefoodchainthroughpaddy.Thenewricevariety,Muktoshri—alsocalled IET21845—,wasdeveloped jointlyby theRiceResearchStationatChinsurahcomingunderWestBengal’sAgricultureDepartmentandtheNationalBotanicalResearchInstitute,Lucknow.Q.80)‘NLRP3’isaproteinwhichwasinnewsrecently.Itisfoundinwhichofthefollowingspecies?

1. Humans2. Bats3. Mice

Selectthecorrectcode:a) 1and2b) 2and3c) 2Onlyd) 1,2and3

Q.80)Solution(d)Itisfoundinalloftheabovespecies.Read More - https://www.thehindu.com/sci-tech/science/how-bats-harbour-several-viruses-yet-not-get-sick/article30711334.eceQ.81)Withreferenceto“WeThinkDigital”,Considerthefollowingstatements:

1. “WeThinkDigital”isGoogleIndia’sdigitalliteracyprogram.

2. Theprogramaims toprovidedigital literacy training toone lakhmenandwomenacrosssevenstates.

IASBABA60DAYPLAN2020–CURRENTAFFAIRSWEEK7AND8

60DAYSPROGRAMME-2020 IASBABA

3. ThesevenstatesareUttarPradesh,Assam,WestBengal,MadhyaPradesh,Gujarat,Jharkhand,andBihar.

Whichofthestatementsgivenaboveis/areCorrect?

a) 1and2only

b) 1and3only

c) 2and3only

d) 1,2,and3

Q.81)Solution(c)

• WeThinkDigital,Facebook’sglobaldigital literacyprogramme,wasannounced in2019duringthesecondeditionofitsSouthAsiaSafetySummit.Theprogramaimstoprovidedigital literacy trainingtoonelakhwomenacrosssevenstates.ThesevenstatesareUttarPradesh,Assam,WestBengal,MadhyaPradesh,Gujarat,Jharkhand,andBihar.

• Uttar Pradeshhas become the first statewhere Facebook Inc, in partnershipwithNationalCommissionforWomen(NCW)andCyberPeaceFoundation,haslaunchedits‘WeThinkDigital’program.

• As part of the program, the company will partner with agencies from bothgovernmentandcivilsocietytodesignlearningmodules.

• Microsoftalsohas theYouthSparkprogramtodevelopdigital skillsofunemployedyouthandmarginalisedwomen.Separately,italsohasaprogramforimprovingskillsofweaversineasternandnortheasternstates.

• GoogleIndia's'InternetSaathi'programstartedin2015,nowcovers2.6lakhvillagesacross20states.

Q.82)Recently,theMoEFCChasissuedanotificationthatseekstobanmembrane-basedwater purification systems (MWPS), primarily reverse osmosis (RO), in areaswhere thewatersuppliedmeetsnorms.Considerthefollowingstatements:

1. TheproblemwithROwaterpurifiersisthatduringthepurificationprocessofwater,around70to80%ofwateriswasted.

2. Thede-mineralizationofwaterisnotrequirediftheinputwaterhasTotalDissolvedSalts(TDS)lessthan500mg/litre.

Whichofthestatementsgivenaboveis/areCorrect?

IASBABA60DAYPLAN2020–CURRENTAFFAIRSWEEK7AND8

60DAYSPROGRAMME-2020 IASBABA

a) 1only

b) 2only

c) Both1and2

d) Neither1nor2

Q.82)Solution(c)

• Recently,theUnionMinistryofEnvironment,ForestandClimateChange(MoEFCC)hasissuedadraftnotificationthatseekstobanmembrane-basedwaterpurificationsystems(MWPS),primarilyreverseosmosis(RO),inareaswherethewatersuppliedmeetsnorms.

• ReverseOsmosis(RO)isawatertreatmentprocessthatremovescontaminantsfromwaterbyusingpressuretoforcemoleculesthroughasemipermeablemembrane.

• PresentROsystemsrecoveronly20percentofwaterwhile80percentgowaste.

• According to BIS standards, drinking water is considered below par if the TDS isabove500mg/l.

• BISstandardsalsostatethatROsystem isnot recommendedfor treatmentof rawwaterhavingArseniclevelabove0.1mg/landFluoridelevelabove8.0mg/l.

• As per a WHO study, TDS levels below 300 mg per litre are considered to beexcellent, while 900 mg per litre is said to be poor and above 1200 mg isunacceptable.

• Thede-mineralizationofwaterisnotrequirediftheinputwaterhasTotalDissolvedSalts(TDS)lessthan500mg/litre.

• IftheTDScountisnotmorethan500mg/liter,andifneededforwaterpurificationsystem,thenUVandUFwaterpurifierarebest.

Q.83)Considerthefollowingstatements

1. Allmedicalequipmentwouldqualifyas“drugs”undertheDrugsandCosmeticsAct,1940.

2. Medicalequipmentunderthisdefinitionincludeimplantablemedicaldevicessuchasknee implants, CT scan, MRI equipment, dialysis machine, PET equipment, X-raymachineetc.

Whichofthestatementsgivenaboveis/areCorrect?

a) 1only

IASBABA60DAYPLAN2020–CURRENTAFFAIRSWEEK7AND8

60DAYSPROGRAMME-2020 IASBABA

b) 2only

c) Both1and2

d) Neither1nor2

Q.83)Solution(c)

• The Union Ministry of Health has notified that from April 2020, all medicalequipmentwouldqualifyas“drugs”undertheDrugsandCosmeticsAct,1940.

• Medicalequipmentunderthisdefinitionincludeimplantablemedicaldevicessuchasknee implants, CT scan, MRI equipment, dialysis machine, PET equipment, X-raymachineetc.

• Asperthenewnotification,alldevices,includinginstrumentsandimplants,whetherused alone or in combination for various purposes like diagnosis, prevention,monitoring,amongothers,willberegulatedunderthelegislation.

• Themanufacture,importandsaleofallmedicaldeviceswillnowneedtobecertifiedbytheCentralDrugsStandardControlOrganisation(CDSCO)

• Itwillhelpgovernmenttoregulatethehighpricesofessentialmedicaldevices.

• CDSCO under Directorate General of Health Services,Ministry of Health & FamilyWelfareistheNationalRegulatoryAuthorityofIndia.

• ItsheadquarterislocatedatNewDelhi.

Q.84)WithreferencetoInsightMission,Considerthefollowingstatements:

1. InSightispartofMarsOrbiterMission-ISRO,India

2. InsightisthefirstmissiondedicatedtolookingdeepbeneaththeMarssurface.

3. InSightmission is the first on the surface ofMars to detectmagnetic signalswithhelpofmagnetometer.

Whichofthestatementsgivenaboveis/areNotCorrect?

a) 1,2and3only

b) 1only

c) 1and3only

d) 2and3only

Q.84)Solution(b)

• InSightispartofNASA’sDiscoveryProgram

IASBABA60DAYPLAN2020–CURRENTAFFAIRSWEEK7AND8

60DAYSPROGRAMME-2020 IASBABA

• InSightmission is the first on the surface ofMars to detectmagnetic signalswithhelpofmagnetometer.

• InsightisthefirstmissiondedicatedtolookingdeepbeneaththeMarssurface.

• InSightmissionfindsthatMarsdoesn’thavetectonicplateslikeEarth.• ThefindingsofMars’formationwillhelpbetterunderstandhowotherrockyplanets,

includingEarth,wereandarecreated

WhyMars?

• Incomparisontotheotherterrestrialplanets,Marsisneithertoobignortoosmall.Thismeansthatitpreservestherecordofitsformationandcangiveusinsightintohowtheterrestrialplanetsformed.

Q.85)Whichofthefollowingstatementsis/areCorrectregardingNipahvirus?

1. Nipah virus (NiV)is a zoonotic virus and can also be transmitted throughcontaminatedfoodordirectlybetweenpeople.

2. Nipah viruswas first identified during an outbreak that took place inMalaysia in1998.

3. AccordingtotheWorldHealthOrganization,currentlytherearenospecificdrugsorvaccinesforNipahvirus.

Selectthecorrectanswerusingthecodegivenbelow:

a) 1,2and3only

b) 1and2only

c) 2only

d) Noneoftheabove

Q.85)Solution(a)

• Nipahvirusisazoonoticvirus(itistransmittedfromanimalstohumans)andcanalsobetransmittedthroughcontaminatedfoodordirectlybetweenpeople.

• FruitbatsofthePteropodidaefamilyarethenaturalhostofNipahvirus.

• In infected people, it causes a range of illnesses from asymptomatic (subclinical)infectiontoacuterespiratoryillnessandfatalencephalitis.Theviruscanalsocauseseverediseaseinanimalssuchaspigs.

• Nipah'snaturalreservoiristhefruitbat,thoughpigshavebeenfoundtobethehostinsomeSoutheastAsianoutbreaks.Theviruscanalsobetransmittedviahuman-to-humancontact.

IASBABA60DAYPLAN2020–CURRENTAFFAIRSWEEK7AND8

60DAYSPROGRAMME-2020 IASBABA

• According to the World Health Organization (WHO), Nipah's case-fatality rate isbetween45%and75%.

• Thenaturalhostsofthevirusarefruitbats.Nipahviruswas first identifiedduringanoutbreakthattookplaceinMalaysiain1998.

• Nipah virus can spread among humans if they establish close contactwithNipah-infectedpeople,batsorpigs.

• AnotherpossiblesourceofinfectioncanbebodiesofpeoplewhodiedduetoNipahvirus.TheNationalCentreforDiseaseControl(NCDC)advisesthathandlingofsuchdeadbodiesshouldbedonestrictlyinaccordancewithgovernmentadvisory.

• People infectedwithNipahvirusmaysufferfrombrainfever;feverwithpersistentcough and difficulty in breathing, acute respiratory infection (mild or severe);influenza-like symptoms -- fever, headache, muscle pain, vomiting, sore throat,dizziness,drowsinessandneurologicalsignsindicatingencephalitis.

• WHO says that in some cases people can also experience pneumonia and severerespiratoryproblems,includingacuterespiratorydistress.

• AccordingtotheWorldhealthOrganisation,currentlytherearenospecificdrugsorvaccinesforNipahvirus.

Q.86)Considerthefollowingstatementswithrespectto‘BilateralNetting’.

1. Bilateralnettingallowstwopartiesinvolvedinaswapagreementtonet-offtheirswappositions.

2. Indianfinancialcontractlawsdonotpermitbilateralnetting.

Selectthecorrectstatementsa) 1Onlyb) 2Onlyc) Both1and2d) Neither1nor2

Q.86)Solution(c)Economic Survey2019-20hasargued in favourof apractice called ‘bilateralnetting’ as awaytoreleasebankcapital.Bilateral netting allows two parties involved in a swap agreement to net-off their swappositions.Suchbilateralnetting,whichistheglobalstandard,willhelpreducetheamountofcapitalyouneedtosetasideforsuchtransactions.According to the survey, Indian financial contract laws do not permit bilateral netting,however, theydo allowmulti-lateral nettingwhereparties canoffset claims against eachotherthroughacentralcounterparty.Without bilateral netting, Indian banks have had to set aside higher capital against theirtrades in the over-the-counter market, which impacts their ability to participate in themarket.Moreover,italsoincreasesthesystemicriskduringdefaults.

IASBABA60DAYPLAN2020–CURRENTAFFAIRSWEEK7AND8

60DAYSPROGRAMME-2020 IASBABA

Bilateralnettingwouldalsohelpreducehedgingcostsandliquidityneedsforbanks,primarydealers and other market-makers, thereby encouraging participation in the over-the-counterderivativesmarket.Itwouldalsohelpdevelopthecorporatedefaultswapsmarket,which,inturn,wouldprovidesupporttothedevelopmentofthecorporatebondmarketTHINK!

• CreditDefaultSwaps(CDS)market

Q.87)Considerthefollowingstatementswithrespectto‘TurantCustoms’.

1. It is aimed at speedy clearance of goods at air and sea ports, which in turn willbenefiteaseofdoingbusinessinIndia.

2. Itisbeinglaunchedby‘TheCentralBoardofDirectTaxes(CBDT)’.

Selectthecorrectstatementsa) 1Onlyb) 2Onlyc) Both1and2d) Neither1nor2

Q.87)Solution(a)TurantCustoms,whichisaimedatspeedyclearanceofgoodsatairandseaports,whichinturnwillbenefiteaseofdoingbusiness in India isbeing launchedbyTheCentralBoardofIndirectTaxesandCustoms(CBIC)Q.88)Consider the followingstatementswith respect to ‘SchemeofAssetManagementandDebtChangeStructure(SAMADHAN)’

1. Underthescheme,astressedpowercompany’sdebtwillbedividedintosustainableandunsustainableportions.

2. The liability of the sustainable debt will be assumed by the new owner and theunsustainablepartwillbeconvertedintoequity.

Selectthecorrectstatementsa) 1Onlyb) 2Onlyc) Both1and2d) Neither1nor2

Q.88)Solution(c)Under the Samadhan scheme, a stressed power company’s debt will be divided intosustainable and unsustainable portions.While the liability of the sustainable debtwill beassumedbythenewowner,theunsustainablepartwillbeconvertedintoequity.Q.89) The ‘Investment Cooperation and Facilitation Treaty’ is in news with respect towhichofthefollowingcountries/organizations?

a) EuropeanUnionb) Brazil

IASBABA60DAYPLAN2020–CURRENTAFFAIRSWEEK7AND8

60DAYSPROGRAMME-2020 IASBABA

c) ASEANd) France

Q.89)Solution(b)Whileapproaching toWTOagainst India forextending support toher sugarcane farmers,Brazilpennedinvestmentcooperationandfacilitationtreaty.ThisisBrazil’s10thandIndia’s4th bilateral investment agreement since both nations had adopted theirModel BilateralInvestmentTreaty.Previously,Indiahasmanagedtoconcludebilateralinvestmenttreatieswith Belarus, Kyrgyzstan, and Cambodia after scrapping down all 83 existing bilateralinvestmenttreaties.Q.90)The‘SingleConvention’whichwasinnewsrecentlydealswith

a) NarcoticDrugsb) RoadSafetyc) WomenSafetyd) OrphanDrugs

Q.90)Solution(a)The Single Convention on Narcotic Drugs of 1961 is an international treaty to prohibitproductionandsupplyofspecific(nominallyhomeotic)drugsandofdrugswithsimilareffectsexceptunderlicenceforspecificpurposes,suchasmedicaltreatmentandresearch.Indiaisaparty.Q.91)WithreferencetoMasalaBonds,Considerthefollowingstatements:

1. MasalabondsarebondsissuedoutsideIndia.

2. Unlikedollarbonds,wheretheinvestorstakethecurrencyrisk,Masalabondmakestheborrowerbeartherisk.

3. TheInternationalFinanceCorporation(IFC)nameditasMasalaBonds.

Whichofthestatementsgivenaboveis/areCorrect?

a) 1,2and3

b) 1and2only

c) 1and3only

d) 2only

Q.91)Solution(c)

• Masala bondsarebondsissued outside India but denominated in Indian Rupees,ratherthanthelocalcurrency.

IASBABA60DAYPLAN2020–CURRENTAFFAIRSWEEK7AND8

60DAYSPROGRAMME-2020 IASBABA

• Masala is an Indian word and it means spices. The term was used bytheInternational Finance Corporation(IFC) to evoke the culture and cuisine ofIndia.

• Unlike dollar bonds, where the borrower takes the currency risk, Masala bondmakestheinvestorsbeartherisk.

• ThefirstMasalabondwasissuedbytheWorldBank-backedIFCinNovember2014whenitraised1,000crorebondtofundinfrastructureprojectsinIndia.

• Later in August 2015 International Financial Cooperation for the first time issuedgreen masala bonds and raised Rupees 3.15 Billion to be used for private sectorinvestmentsthataddressclimatechangeinIndia.

• InJuly2016HDFCraised3,000crorerupeesfromMasalabondsandtherebybecamethefirstIndiancompanytoissuemasalabonds.

• In themonth of August 2016 public sector unitNTPCissued first corporate greenmasalabondsworth2,000crorerupees.

Q.92)Considerthefollowingstatements

1. Budget 2020 provision seeks to tax income of non-resident Indians who are notpayingtaxesinanyothertaxjurisdictions.

2. Incomeearnedoutside Indiabyhimshallnotbe taxed in Indiaunless it isderivedfromanIndianbusinessorprofession.

Whichofthestatementsgivenaboveis/areCorrect?

a) 1only

b) 2only

c) Both1and2

d) Neither1nor2

Q.92)Solution(c)

• Budget2020amendedthetaxresidencyrulesforNRIs.Tillfinancialyear(FY)2019-20,anNRIwhovisitedIndiawouldbeconsideredaresidentiftheyspent182daysormoreinthepreviousyearinthecountry,inadditiontoanaggregatestayof365daysormoreintheprecedingfouryears.Budget2020proposedtolowerthethresholdperiodofstayinthepreviousyearto120daysfrom182days.

• Thebudgetproposedtoamendthedefinitionof“notordinarilyresident".TillFY20,anindividualwasclassifiedasa“notordinarilyresident"ifhewasanon-residentin

IASBABA60DAYPLAN2020–CURRENTAFFAIRSWEEK7AND8

60DAYSPROGRAMME-2020 IASBABA

India for nine out of 10 preceding years. The budget proposal has reduced thenumbersofyearstosevenoutofthe10precedingyears

• The government is seeking to tax NRIs who are carrying on substantial economicactivitiesfromIndia.Underthepresentresidencecriteriaofaminimumstayof182daysinanFY,NRIsremainnon-residentinIndiaperpetually.

• Individuals who are non-residents of all the countries in which they work but acitizenofIndiawouldbedeemedtobeIndiantaxresidents.

• Budget 2020 provision seeks to tax income of non-resident Indians who are notpayingtaxesinanyothertaxjurisdictions.

• Incomeearnedoutside Indiabyhimshallnotbe taxed in Indiaunless it isderivedfromanIndianbusinessorprofession.

Q.93)WithreferencetoNationalWarMemorial,Considerthefollowingstatements:

1. TheNationalWarMemorialdrawsinspirationfromthe'Chakravyuh'formation

2. TheNationalWarMemorialpaystributetosoldierswholaiddowntheirlivesonlyinwarsagainstPakistanandChina.

Whichofthestatementsgivenaboveis/areCorrect?

a) 1only

b) 2only

c) Both1and2

d) Neither1nor2

Q.93)Solution(a)

• IndiaGate itself isawarmemorialbuiltduring theBritishRajas theAll IndiaWarMemorialArchtohonourthesoldierswhodiedintheFirstWorldWar(1914-1918)andtheThirdAnglo-AfghanWar(1919).

• The National War Memorial pays tribute to soldiers who laid down their livesdefending the nation during the India-China war in 1962, Indo-Pak wars in 1962,Indo-Pakwars in 1947, 1965 and 1971, Indian Peace Keeping ForceOperations inSrilankaandintheKargilconflictof1999.

• TheNationalWarMemorialalsocommemoratesthesoldierswhoparticipatedandmade the supreme sacrifice in Peace Keeping Missions, and Counter InsurgencyOperations.

• TheNationalWarMemorialdrawsinspirationfromthe'Chakravyuh'formation

IASBABA60DAYPLAN2020–CURRENTAFFAIRSWEEK7AND8

60DAYSPROGRAMME-2020 IASBABA

• The memorial comprises four concentric circles, namely Amar Chakra, VeertaChakra,TyagChakraandRakshakChakra.

• TheRakshakChakra theoutermostonecomprisesof rowsofmore than600 treeswitheachtreerepresentingmanysoldierswhoguardtheterritorialintegrityofthenationroundtheclock.

AmarJawanJyotiissymbolisedbyaninvertedbayonetandsoldier'shelmetoveritwithaneternalflameburningbesideit.Itwasbuiltin1972underneaththeIndiaGatearchtocommemoratesoldiersmartyredintheIndo-PakWarof1971.Q.94) Which of the following statements is/are Correct regarding Parambikulam TigerReserve?

1. ParambikulamTigerReserveisaprotectedareainTamilNadu.

2. The sanctuary is the homeof four different tribesof indigenous peoples includingtheKadar,Malasar,MuduvarandMalaMalasar.

3. Theoldesteverteaktree,KannimaraTeakexistshere.

Selectthecorrectanswerusingthecodegivenbelow:

a) 1and2only

b) 1only

c) 2and3only

d) 1,2and3

Q.94)Solution(c)

ParambikulamTigerReserveisaprotectedareainPalakkaddistrictofKeralastate.

• ItisintheSungamrangeofhillsbetweentheAnaimalaiHillsandNelliampathy

• ParambikulamWildlife Sanctuarywas declared as part of the Parambikulam TigerReserveonFebruary19,2010.

• The sanctuary is the homeof four different tribesof indigenous peoples includingtheKadar,Malasar,MuduvarandMalaMalasar.

• Parambikulam Tiger Reserve implements theParticipatory Forest ManagementScheme(PFMS).

• Theoldesteverteaktree,KannimaraTeakexistshere.Itisabout450years.ItwontheMahavrikshaPuraskargivenbytheIndianGovernment.

Q.95)WithreferencetoSharang,Considerthefollowingstatements:

IASBABA60DAYPLAN2020–CURRENTAFFAIRSWEEK7AND8

60DAYSPROGRAMME-2020 IASBABA

1. The Sharang has been upgraded successfully from its previous specification byBharatDynamicsLimited.

2. SharangisanAnti-TankGuidedMissile.

Whichofthestatementsgivenaboveis/areCorrect?

a) 1only

b) 2only

c) Both1and2

d) Neither1nor2

Q.95)Solution(d)

• The Sharang has been upgraded successfully from its previous specification byOrdnanceFactoryBoard.

• The Indigenously upgraded 155mm/45 calibre artillery gun, Sharang gun systemwas successfully test-fired at the Long Proof Range (LPR) of Khamaria in JabalpurdistrictofMadhyaPradesh

• Shellsfiredfromthistowedartilleryguncan,ittargetsatadistanceof39km.

• Theindigenously-designedgunispartofthegovernment’s‘MakeinIndia’initiative,andthe Indianarmywillbesupplied fromGunCarriageFactoryandtheOrdnanceFactory,Kanpur.

• TheSharanggunhasbeenupgraded successfully from itsprevious specificationof130mmto155mmanditscommercialproductionwillbegansoon.

• TheVehicleFactoryJabalpur(VFJ)willassemble12of30Sharanggunsystemsandtheother18gunswillbeassembledatGunCarriageFactory,Kanpur.

• Sharanggunsystemsareequippedwithnightvisionsensorstotargettheenemyandcanbeoperatedatnight.

• Sharang gun has been named after the celestial bowSharangacrafted byVishwakarmawhohadgivenittoLordVishnu.

Q.96)Considerthefollowingstatements

1. ‘Bengal Eastern Frontier Regulations (BEFR), 1873’ empowered the LieutenantGovernortodefineaninnerline,beyondwhichnoBritishsubjectofcertainclassesorforeignresidentscouldpasswithoutalicence.

2. The Government of India Act, 1935 prescribed “excluded areas” and “partiallyexcluded areas” to be under the direct control of the provincial governor, to theexclusionoftheIndianlegislature.

IASBABA60DAYPLAN2020–CURRENTAFFAIRSWEEK7AND8

60DAYSPROGRAMME-2020 IASBABA

Selectthecorrectstatementsa) 1Onlyb) 2Onlyc) Both1and2d) Neither1nor2

Q.96)Solution(c)TheBEFR empowered the LieutenantGovernor to define an inner line, beyondwhich noBritishsubjectofcertainclassesorforeignresidentscouldpasswithoutalicence,givingthegovernment untrammelled control. These regulations, which are still in force, coverArunachalPradesh,MizoramandNagaland.ManipurhasalsonowbeensubsumedintothepurviewoftheInnerLine.TheGovernmentof IndiaAct,1935,wentastepfurther,prescribing“excludedareas”and“partiallyexcludedareas”tobeunderthedirectcontroloftheprovincialgovernor,totheexclusionoftheIndianlegislature.Q.97)‘YongleBlueHole’wasinnewsrecently.Whereisitlocated?

a) EastPhilippinesSeab) YellowSeac) BismarckSead) SeaofAzov

Q.97)Solution(a)Yongle Blue Hole is the world’s deepest blue hole. It is located in South China Sea/EastPhilippinesSea.Ithasadepthof300metres, fardeeper than thepreviously recordeddeepestbluehole,Dean’sBlueHoleinBahamas,whichhadadepthof202metres.Likemostblueholes,it isanoxic.

IASBABA60DAYPLAN2020–CURRENTAFFAIRSWEEK7AND8

60DAYSPROGRAMME-2020 IASBABA

Q.98)WhichofthefollowingcountriesisIndia’slargesttradingpartner?

a) UnitedArabEmiratesb) Chinac) USAd) UnitedKingdom

Q.98)Solution(c)The US has surpassed China to become India's top trading partner, showing greatereconomictiesbetweenthetwocountries.Q.99)Whichofthefollowingpairsarecorrectlymatched?

1. EaglenestWildlifeSanctuary–Assam2. MalaiMahadeshwaraWildlifeSanctuary–Karnataka3. TholpettyWildlifeSanctuary–Kerala

Selectthecorrectcode:a) 1and2b) 3Onlyc) 2and3d) 1,2and3

Q.99)Solution(c)EaglenestWildlifeSanctuary–ArunachalPradeshMalaiMahadeshwaraWildlifeSanctuary–KarnatakaTholpettyWildlifeSanctuary–KeralaQ.100)Whichofthefollowingcountrieshavecollaboratedwith Indiaforagovernment-to-governmentagreementonjetenginetechnologydevelopment?

IASBABA60DAYPLAN2020–CURRENTAFFAIRSWEEK7AND8

60DAYSPROGRAMME-2020 IASBABA

a) Franceb) UnitedKingdomc) Japand) Israel

Q.100)Solution(b)TheanswerisUK.Read More - https://www.thehindu.com/news/national/india-uk-to-sign-pact-on-jet-engine-development/article30755415.eceQ.101)Considerthefollowingstatements:

1. The Artificial Intelligence based ASKDISHA chatbot is developed to registercomplaintsofwomenwhichareharassmentrelated.

2. TheASKDISHAChatbotcanconverseeveninHindilanguage.

Whichofthestatementsgivenaboveis/areCorrect?

a) 1only

b) 2only

c) Both1and2

d) Neither1nor2

Q.101)Solution(b)

• Artificial IntelligencebasedASKDISHAchatbot isdeveloped to resolvequeriesofrailway passengers over the internet pertaining to various services offered byIRCTC

• In order to resolve queries of railway passengers over the internet pertaining tovarious services offered, Indian Railways had introduced the services of ArtificialIntelligencebasedASKDISHAchatbot inOctober2018 for thebenefitof theusers,IndianRailwaysCatering&TourismCorporationLimited(IRCTC).

• The ASKDISHA Chatbot was initially launched in English language. IRCTC has nowpoweredvoice enabledASKDISHA to conversewith customers inHindi languagealso.ThecustomerscannowaskqueriestoASKDISHAinHindilanguagebyvoiceaswellastext.

• IRCTCplanstolaunchASKDISHAinmorelanguagesalongwithmanyotheradditionalfeaturesinthenearfuture.

• The chatbot is a special computer programme designed to simulate conversationwithusers,especiallyovertheinternet.

IASBABA60DAYPLAN2020–CURRENTAFFAIRSWEEK7AND8

60DAYSPROGRAMME-2020 IASBABA

Q.102)WithreferencetoOneHealth,Considerthefollowingstatements:

1. TheconceptofOneHealthrecognizesthathealthofhumanbeingsisconnectedtohealthofanimalsandenvironment.

2. TheconceptofOneHealthcanbeeffectivelyimplementedforreducingincidenceofemergingzoonoticthreats.

Whichofthestatementsgivenaboveis/areCorrect?

a) 1only

b) 2only

c) Both1and2

d) Neither1nor2

Q.102)Solution(c)

• The concept of ‘One Health’, which recognises that health of human beings isconnectedtohealthofanimalsandenvironment,isgainingimportanceasmostofthe contagious diseases affecting humans are zoonotic (animal to man origin) innature.

• “TheconceptofOneHealthcanbeeffectively implementedforreducing incidenceofemergingzoonoticthreatslikeCOVID-19.

• Of the contagious diseases affecting humans, more than 65% are of zoonotic oranimaltomanorigin.

• The Kerala Veterinary and Animal Sciences University (KVASU) has launched OneHealth centre to address the emerging zoonotic threats ike avian influenza, swineflu,monkeyfever,coronavirusinfection,etc.

• Increasing stress on animals due to loss of their habitat would increase scope ofzoonoticdiseases.

Q.103) Which of the following statements is/are Correct regarding Atal Bhujal Yojana(ABHY)?

IASBABA60DAYPLAN2020–CURRENTAFFAIRSWEEK7AND8

60DAYSPROGRAMME-2020 IASBABA

1. Itisacentralsectorscheme.

2. ItisbeingimplementedbytheMinistryofJalShakti.

3. HalfofthecostoftheschemewillbebornebytheGovernment,whiletheotherhalfwillbefundedbyUNWater.

Selectthecorrectanswerusingthecodegivenbelow:

a) 1,2and3only

b) 1and3only

c) 2only

d) 1and2only

Q.103)Solution(d)

• The Atal Bhujal Yojana (ABHY) is acentral sector scheme worthRs.6,000croreforsustainablemanagementofgroundwaterwithcommunityparticipation.

• It envisages people's participation through the formation of ‘Water UserAssociations’,waterbudgeting,preparation& implementationofGram-panchayat-wisewatersecurityplans,etc.

• It isbeing implementedby theMinistry of Jal Shakti(earlier knownasMinistryofWaterResources,RiverDevelopmentandGangaRejuvenation).

• The objective of the programme was to lay emphasis on the recharge ofgroundwaterresourcesandimprovetheexploitationofthegroundwaterresources,withtheinvolvementofpeopleatthelocallevel.

• HalfofthecostoftheschemewillbebornebytheGovernment,whiletheotherhalfwillbefundedbytheWorldBankintheformofaloan.

Q.104) Which of the following statements is/are correct regarding Narcotics ControlBureau(NCB)?

1. Itenforcestheprohibitionofthesmugglingofitemsincludingdrugs,gold,diamonds,electronics,foreigncurrency,andcounterfeitIndiancurrency.

2. TheNCB’snationalheadquartersislocatedinMumbai.

3. ItcomesunderMinistryofMinistryofHomeAffairs.

Selectthecorrectanswerusingthecodegivenbelow:

a) 1and3only

b) 1only

c) 2and3only

IASBABA60DAYPLAN2020–CURRENTAFFAIRSWEEK7AND8

60DAYSPROGRAMME-2020 IASBABA

d) 1,2,and3

Q.104)Solution(c)

DirectorateofRevenue Intelligence (DRI) is themajor intelligenceagencywhichenforcesthe prohibition of the smuggling of items including drugs, gold, diamonds, electronics,foreigncurrency,andcounterfeitIndiancurrency.

NarcoticsControlBureau

• ItwasconstitutedbytheGovernmentofIndiain1986undertheNarcoticDrugsandPsychotropicSubstancesAct,1985.

• Theactprovidesforthepenaltyofpropertyderivedfromorusedinillegaltrafficinnarcoticdrugs.

• ItistheapexdruglawenforcementandintelligenceagencyresponsibleforfightingdrugtraffickingandtheabuseofillegalsubstancesinIndia.

• TheNarcotics Control Bureau's national headquarters is located inDelhi and it isaffiliatedtoMinistryofHomeAffairs.

Functions:

• Coordination among various Central and State Agencies engaged in drug lawenforcement;

• AssistingStatesIenhancingtheirdruglawenforcementeffort;

• Collectionanddisseminationofintelligence;

• Analysisofseizuredata,studyoftrendsandmodusoperandi;

• PreparationofNationalDrugEnforcementStatistics;

• Liaison with International agencies such as UNDCP, INCB, INTERPOL, CustomsCooperationCouncil,RILOetc;

• Nationalcontactpointforintelligenceandinvestigations

Q.105)Theterm“OperationVanilla”issometimesseeninthenewswithreferenceto:

a) It aims to unify the islands of Comoros, Mauritius, Madagascar, La Reunion,SeychellesMaldivesandMayotte,underonetourismbrandthroughtheaffiliationofeachoftheislands.

b) It istheoperation launchedby IndianNavytoassistthepopulationofMadagascaraffectedbyCycloneDiane.

c) It is the operation against fighting the vanilla thieves ofMadagascar launched bygovernmentofMadagascar.

IASBABA60DAYPLAN2020–CURRENTAFFAIRSWEEK7AND8

60DAYSPROGRAMME-2020 IASBABA

d) ItisanoperationofIndianAirForcetobringbacktheIndiansstuckinChinaduetoCovid19

Q.105)Solution(b)

• Recently, the Indian Navy has launched the 'Operation Vanilla' to assist thepopulationofMadagascaraffectedbyCycloneDiane.

• Indian Navy also diverted large amphibious ship viz.INS Airavatcarrying clothing,medicines,andotherreliefmaterial toprovideassistanceandsupport in thereliefoperations.

CycloneDiane

• Itisatropicalcyclone.

• Origin:North-westoffthecoastofMauritiusintheSouth-westernIndianOcean.

• Madagascar has been hit by it and there has been heavy flooding and landslipscausinglossoflivesanddisplacement,affectingmorethan92,000people.IndiahasbeenthefirstcountrytorespondtofloodsinMadagascar.

Q.106)Considerthefollowingstatementswithrespectto‘GandhinagarDeclaration’.

1. The Declaration urges Parties and other governments to ensure effective liaisonbetweenthenationalfocalpointsoftheConventiononMigratorySpeciesandthoseof the Convention on Biodiversity and other biodiversity-related conventions andagreements, aswell as the ParisAgreement under theUnitedNations FrameworkConventiononClimateChange.

2. The Declaration recognizes the importance of synergies and cooperation amongbiodiversity-related conventions andothermultilateral environmental agreements,and that their role should be clearly reflected in the post-2020 global biodiversityframework.

Selectthecorrectstatementsa) 1Onlyb) 2Only

IASBABA60DAYPLAN2020–CURRENTAFFAIRSWEEK7AND8

60DAYSPROGRAMME-2020 IASBABA

c) Both1and2d) Neither1nor2

Q.106)Solution(c)TheGandhinagarDeclarationemphasizesthatimprovementofecologicalconnectivityisthetop priority for the Convention onMigratory Species in the post-2020 global biodiversityframeworkandthattheroleoftheConventionbodyshouldbeclearlyreflectedinthepost-2020framework.The Declaration also recognizes the importance of synergies and cooperation amongbiodiversity-relatedconventionsandothermultilateralenvironmentalagreements,andthattheirroleshouldbeclearlyreflectedinthepost-2020globalbiodiversityframework.TheDeclarationurgesParties andother governments toensureeffective liaisonbetweenthe national focal points of the Convention on Migratory Species and those of theConventiononBiodiversityandotherbiodiversity-relatedconventionsandagreements,aswellas theParisAgreementunder theUnitedNationsFrameworkConventiononClimateChange. This is with a view to reflect the respective priorities of, and align their effortsunderthevariousagreementsrelatedtothepost-2020globalbiodiversity frameworkandnationalbiodiversitystrategiesandactionplans.Anotherkeyoutcomeofthemeetingwastheadditionof10newspeciestotheConventionon Migratory Species Appendices, including Asian elephants, jaguars, and great Indianbustards, Bengal floricans, little bustards, antipodean albatrosses and oceanic white-tipsharks,allslatedtoreceivethestrictestprotectionunderAppendixI.Q.107)AccordingtoFAO,whichofthefollowingareasareidentifiedashotspotsofthreateninglocustactivity?

1. HornofAfrica2. RedSeaArea3. SouthwestAsia4. PearlRiverDelta

Selectthecorrectcode:a) 1,2and3b) 2,3and4c) 1,3and4d) 1,2and4

Q.107)Solution(a)TheFoodandAgricultureOrganisation(FAO)oftheUnitedNationshascurrentlyidentifiedthreehotspotsofthreateninglocustactivity,wherethesituationhasbeencalled“extremelyalarming”—theHornofAfrica,theRedSeaarea,andsouthwestAsia.TheHornofAfricahasbeencalledtheworst-affectedarea,wheretheFAOhassaidthereis“anunprecedentedthreattofoodsecurityandlivelihoods”.FourspeciesoflocustsarefoundinIndia:

• Desertlocust(Schistocercagregaria),• Migratorylocust(Locustamigratoria),

IASBABA60DAYPLAN2020–CURRENTAFFAIRSWEEK7AND8

60DAYSPROGRAMME-2020 IASBABA

• BombayLocust(Nomadacrissuccincta)and• Treelocust(Anacridiumsp.).

Q.108)Whichofthefollowingstatementsis/arecorrect?

1. Hundredpercent incometaxexemption isgivenfromprofitderivedfromactivitiessuch as post-harvest value addition to agriculture by Farmer Producers'Organisations(FPO)’shavingannualturnoveruptoRs100crores.

2. Hundred percent FDI is permitted under the approval route in Food ProcessingIndustriesandfortyninepercentFDIisallowedthroughautomaticroutefortradingincluding e-commerce in respect of food products manufacture and produced inIndia.

Selectthecorrectstatementsa) 1Onlyb) 2Onlyc) Both1and2d) Neither1nor2

Q.108)Solution(a)HundredpercentFDIispermittedundertheautomaticrouteinFoodProcessingIndustriesand100percentFDIisallowedthroughapprovalroutefortradingincludinge-commerceinrespectoffoodproductsmanufactureandproducedinIndia.Further,100percentincometax exemption is given from profit derived from activities such as post-harvest valueadditiontoagriculturebyFPO’shavingannualturnoveruptoRs100crores.Q.109) ‘India Economic Strategy to 2035 (IES 2035)’ is associated with which of thefollowingcountries?

a) USAb) Australiac) Japand) France

Q.109)Solution(b)The India Economic Strategy is an ambitious plan to transform Australia's economicpartnershipwithIndiaoutto2035.Q.110)‘NationalTigerConservationAuthority(NTCA)’issetupunder

a) WildlifeProtectionAct,1972b) Environment(Protection)Act,1986c) BiologicalDiversityAct,2002d) Noneoftheabove

Q.110)Solution(a)

IASBABA60DAYPLAN2020–CURRENTAFFAIRSWEEK7AND8

60DAYSPROGRAMME-2020 IASBABA

The National Tiger Conservation Authority is a statutory body under the Ministry ofEnvironment, Forests and Climate Change constituted under enabling provisions of theWildlife(Protection)Act,1972,asamendedin2006,forstrengtheningtigerconservation,asperpowersandfunctionsassignedtoitunderthesaidAct.The National Tiger Conservation Authority (NTCA) was established in December 2005followingarecommendationoftheTigerTaskForce,constitutedbythePrimeMinisterofIndiaforreorganisedmanagementofProjectTigerandthemanyTigerReservesinIndia.Q.111)Whichofthefollowingstatementsis/areNotCorrectregardingGramNyayalayas?

1. TheStateGovernment,inconsultationwiththeHighCourt,notifiestheboundariesoftheareaunderthejurisdictionofaGramNyayalaya.

2. TheStateGovernmentshallappointapresidingofficercalledNyayadhikariforeveryGramNyayalayainconsultationwiththeDistrictCourt.

3. GramNyayalayashallexerciseonlyciviljurisdiction.

Selectthecorrectanswerusingthecodegivenbelow:

a) 1and2only

b) 2and3only

c) 3only

d) 1,2and3

Q.111)Solution(b)

• TheStateGovernment,inconsultationwiththeHighCourt,notifiestheboundariesoftheareaunderthejurisdictionofaGramNyayalaya.

• TheStateGovernmentshallappointapresidingofficercalledNyayadhikariforeveryGramNyayalayainconsultationwiththeHighCourt.

• GramNyayalayashallexercisebothcivilandcriminaljurisdiction.• The judgment passed by aGramNyayalaya in civil cases shall be deemed to be a

decree.• GramNyayalayaisestablishedforeveryPanchayatatintermediateleveloragroup

ofcontiguousPanchayatsatintermediatelevelinadistrict.

• 114th Report of the Law Commission (1986) recommended setting up of GramNyayalayas(mobilevillagecourts)atthegrassrootlevels

• ThefirstandsecondschedulesoftheGramNyayalayaActcanbeamendedbyboththecentralandstategovernments.

IASBABA60DAYPLAN2020–CURRENTAFFAIRSWEEK7AND8

60DAYSPROGRAMME-2020 IASBABA

• A Gram Nyayalaya is not bound by the rules of evidence provided in the IndianEvidenceAct,1872butisguidedbytheprinciplesofnaturaljustice

• An appeal against a judgement of a criminal case shall be taken to the Court ofSession,whileacivilcaseappealshallbetakentoDistrictcourt.

Q.112)Whatisthemainaimof‘GenomeIndia’Project?

a) To vastly add to the available informationon thehuman species andadvance thecause,bothbecauseofthescaleoftheIndianpopulationandthediversityhere.

b) To decode of the entire human genome as done by an international programme-HumanGenomeProject(HGP1990-2003).

c) ToultimatelybuildagridoftheIndian“referencegenome”,tounderstandfullythetypeandnatureofdiseasesandtraitsthatcomprisethediverseIndianpopulation.

d) Tocreateadatabaseofgeneticinformationandgenemodification.

Q.112)Solution(c)

• Genome India Project willinvolve 20 leading institutions including the IndianInstituteofScience(IISc)inBengaluruandafewIITs.

• The first stageof theprojectwill lookat samplesof“10,000persons fromalloverthe country” to form a “grid” that will enable the development of a “referencegenome”.

• TheIISc’sCentreforBrainResearch,anautonomousinstitute,willserveasthenodalpointoftheproject.

• Themain aim of GIP ultimately build a grid of the Indian “reference genome”, tounderstandfullythetypeandnatureofdiseasesandtraitsthatcomprisethediverseIndianpopulation.

• GIP aims to vastly add to the available information on the human species andadvance the cause, both because of the scale of the Indian population and thediversityhere.

• The Genome India Project is inspired by theHuman Genome Project (HGP 1990-2003)- an international programme that led to the decoding of the entire humangenome.

Q.113) With reference to Inner Line Permit (ILP) and Protected Area Permit (PAP),Considerthefollowingstatements:

IASBABA60DAYPLAN2020–CURRENTAFFAIRSWEEK7AND8

60DAYSPROGRAMME-2020 IASBABA

1. No permits (ILP and PAP) are required to visitAssam,Meghalayaand Tripura forbothIndianNationalsandforeigners.

2. NagalandistherecentstatetojointheILPstatelist.

3. ProtectedAreaPermitisrequiredforvisitingcertainpartsofUttarakhand.

Whichofthestatementsgivenaboveis/areNotCorrect?

a) 1,2and3only

b) 2and3only

c) 2only

d) 1only

Q.113)Solution(c)

• Manipur is the last state to join the ILP state list andManipur has launched anonlineportalforthetravellerstoseekInnerLinePermits.

• No permits (ILP and PAP) are required to visitAssam,Meghalayaand Tripura forbothIndianNationalsandforeigners.

• ForeignnationalscanapplyforILPandPAPiftheyhaveanE-VisaforIndia.ItisnotmandatorytohavearegularTouristVisaforapplyingpermitsforNortheastIndiaforforeignnationals.

• Overseas Citizen of India card holders are classified as foreigners and they mustobtainpermitstovisitNorthEasternstates.

• AnILPisissuedbythestategovernmentconcerned.

Q114)Considerthefollowingstatementsregardinglocustswarms

1. DesertLocustsareonlyfoundinAfrica.

2. Inasingleday,alocustswarmcantravelnearly100milesandeatitsownweightinweightinfreshfood.

3. Locustscarrydiseasesthatcanharmhumans.

Whichofthestatementsgivenaboveis/areNotCorrect?

a) 1,2and3only

b) 1only

c) 1and3only

d) 2only

IASBABA60DAYPLAN2020–CURRENTAFFAIRSWEEK7AND8

60DAYSPROGRAMME-2020 IASBABA

Q.114)Solution(c)

• Major locust attacks have been observed in several countries in western andsouthernAsiaandineasternAfrica.

• Four species of locusts are found in India: Desert locust (Schistocerca gregaria),Migratory locust (Locusta migratoria), Bombay Locust (Nomadacris succincta) andTreelocust(Anacridiumsp.).

• DesertLocustsareusuallyrestrictedtothesemi-aridandariddesertsofAfrica,theNearEastandSouth-WestAsiathatreceivelessthan200mmofrainannually.

• The locust swarms had damaged 100% of Somalia’s staple crops of maize andsorghum,accordingtotheFoodandAgricultureOrganizationoftheUnitedNations.

• Locustsdoesnotcarryanydiiseasesthatareknowntoharmhumans.

• InKenya,upto30%ofpasturelandhasbeenlost.

• LocustshavegorgedoncropsinSouthSudan,alreadyreelingfromyearsofcivilwarandwidespreadhunger.

• Theyhave laidneweggs in Ethiopia, Eritrea,Djibouti andUganda. TheU.N. is stilltryingtoassessthedamageinmanyplaces.

• Inasingleday,alocustswarmcantravelnearly100milesandeatitsownweightinleaves,seeds,fruitsandvegetables—asmuchas35,000peoplewouldconsume.Atypicalswarmcanstretchover30squaremiles.

Q.115)PakkeTigerReserveissituatedinwhichofthefollowingstate:a) Assamb) ArunanchalPradeshc) Nagalandd) Manipur

Q.115)Solution(b)

• PakkeTigerReserveissituatedinthestateofArunanchalPradesh• Corridor,thehighwayaimstoconnectArunachalPradeshwithAssam.• Pakke Tiger Reserve (declared in 1999 - 2000) lies in the foothills of the eastern

HimalayaintheEastKamengdistrictofArunachalPradesh.ItisalsoknownasPakhuiTigerReserve.

• ItfallswithintheEasternHimalayaBiodiversityHotspot.• Itisknownforitsamazingsightingsoffourresidenthornbillspecies• Itisboundedbykamengriverinthewestandnorthandbypakkeriverintheeast.

Q.116)Whichof the followingstatements is/arecorrectwithrespect to ‘Nutrient-basedsubsidy(NBS)’scheme?

1. Afixedamountofsubsidy,decidedonanannualbasis,isprovidedoneachgradeofsubsidisedphosphaticandpotassic(P&K)fertilisers.

IASBABA60DAYPLAN2020–CURRENTAFFAIRSWEEK7AND8

60DAYSPROGRAMME-2020 IASBABA

2. A complex fertilizer, ammonium phosphate, was recently included under thenutrient-basedsubsidyscheme.

Selectthecorrectstatementsa) 1Onlyb) 2Onlyc) Both1and2d) Neither1nor2

Q.116)Solution(c)In2010,thegovernmenthadlaunchedthenutrient-basedsubsidy(NBS)programmeunderwhichafixedamountofsubsidy,decidedonanannualbasis,isprovidedoneachgradeofsubsidisedphosphaticandpotassic(P&K)fertilisers,exceptforurea,basedonthenutrientcontentpresentinthem.Retail prices of non-urea fertilisers such as Di-ammonium Phosphate (DAP), Muriate ofPotash(MoP)andNPKaredecontrolledandaredeterminedbymanufacturers,whileCentregivesafixedsubsidyeachyear.Recently,CCEAalsoapprovedthe inclusionofacomplex fertilizer,ammoniumphosphate,underthenutrient-basedsubsidyscheme.Q.117)Whichofthefollowingstatements

1. Thenumbersofvulturesandeagleshavedecreasedinthecountry.2. ThefourspeciesofbustardsinIndiahaveallsufferedcontinuouspopulationdeclines

becauseofhistoricalhuntingandwidespreadhabitatloss.3. ThenumberofbirdsintheWesternGhatshavesignificantlyincreasedsince2000.

Selectthecorrectstatementsa) 1Onlyb) 1and2c) 1and3d) 1,2and3

Q.117)Solution(b)ThenumberofbirdsintheWesternGhats,whichisconsideredoneoftheworld’sforemostbiodiversityhotspots,alsodeclinedbyalmost75percentsince2000.Thenumbersofvulturesandeagleshavedecreasedinthecountry.ThefourspeciesofbustardsinIndia–theGreatIndianBustard,Macqueen’sBustard,LesserFlorican andBengal Florican, have all suffered continuouspopulationdeclinesbecauseofhistorical hunting and widespread habitat loss, compounded with their slow growth andreproduction.Q.118)Whichofthefollowingpairsarecorrectlymatched?

1. Kalasa-BanduriNalaProject–Karnataka2. KaleshwaramLiftIrrigationProject–Maharashtra3. PaithanHydroelectricPowerPlant–Telangana

IASBABA60DAYPLAN2020–CURRENTAFFAIRSWEEK7AND8

60DAYSPROGRAMME-2020 IASBABA

Selectthecorrectcode:a) 1Onlyb) 1and2c) 2and3d) 1,2and3

Q.118)Solution(a)Kalasa-BanduriNalaProject–KarnatakaKaleshwaramLiftIrrigationProject–TelanganaPaithanHydroelectricPowerPlant–MaharashtraQ.119)The‘Neur’and‘Dinka’communitieswereinnrwsrecently.Theybelongto

a) Meghalayab) Ladakhc) Mizoramd) Noneoftheabove

Q.119)Solution(d)Theybelongto‘SouthSudan’.Q.120)Whichofthefollowingnationalparkslostabout650sq.km.intwodecadesfrom1991?

a) BandipurNationalParkb) PapikondaNationalParkc) OrangNationalParkd) RanthamboreNationalPark

Q.120)Solution(b)Bysomeestimates,theEasternGhatshaveshrunkby16%overthepastcentury,andjustoneregion,PapikondaNationalPark,lostabout650sq.km.intwodecadesfrom1991.Source: https://www.thehindu.com/opinion/editorial/a-browning-east-the-hindu-editorial-on-climate-change-impact-on-the-eastern-ghats/article30935306.ece

Copyright©byIASbabaAll rights are reserved.Nopart of this documentmaybe reproduced, stored in a retrievalsystemor transmitted inany formorbyanymeans,electronic,mechanical,photocopying,recordingorotherwise,withoutpriorpermissionofIASbaba.

IASBABA60DAYPLAN2020–CURRENTAFFAIRSWEEK7AND8

60DAYSPROGRAMME-2020 IASBABA